SlideShare a Scribd company logo
See discussions, stats, and author profiles for this publication at: https://www.researchgate.net/publication/266738818
A Handbook of MCQs in Toxicology
Book · May 2014
CITATIONS
0
READS
50,587
All content following this page was uploaded by Wajhul Qamar on 25 November 2019.
The user has requested enhancement of the downloaded file.
Preface
The goal of this book is to provide comprehensive multiple choice
questions from the field of toxicological science. It also includes an
intention to provide a brain storming session for the individuals
associated with toxicology. The present book “A Handbook of MCQs in
Toxicology” is targeted towards students and general audiences. This
book provides an ample number of toxicological questions for the
students preparing for their examinations or simply for testing and
practicing knowledge of toxicology. The book will also help those who
want to improve their toxicological knowledge in a smaller amount of
time. With the best wishes this book is presented to those interested in
toxicology.
Wajhul Qamar, Ph.D.
Assistant Professor
Research Center,
College of Pharmacy
King Saud University
Riyadh, Saudi Arabia
Dedicated to my family
"All substances are poisons: there is none which is not a
poison. The right dose differentiates a poison and a
remedy."
Paracelsus (1493-1541)
Introduction
Every living entity has an interaction with other livings or
non-livings for their appropriate survival. These kinds of
interactions are essential as organisms are dependent on other
living and non-living matter to obtain energy and achieve growth
so that they could fulfill all the requirements to be alive. Non-
living matter play an irreplaceable role in normal physiology of
the living beings. Various enzymes play their essential role only in
the presence of a cofactor, which are mainly inorganic metal ions
including Mg++, Mn++, Zn++ etc. Ca++ also plays an essential role in
cellular signaling and bone formation; these are to mention a few.
Other components that are involved in physiology include
selenium, sodium, potassium, iron etc. and organisms require all
these elements on a daily basis through food and water. Oxygen is
also an essential element that play major role in oxidative
metabolism and generation of energy for the daily need of the
organism.
But all of above mentioned or other components not mentioned,
are needed in a particular dose1. Uptake of these components
above or below the required dose can potentially alter the
physiology and exhibit a condition that is a disease or ailment. In
the same manner a substance that is not required to the organism
can have adverse physiological alterations and severe effects after
an exposure. These unwanted substances mainly include a variety
of chemicals; others are biological (e.g. viruses, bacteria etc.) or
physical (e.g. UV radiations) agents. Agents that impose such kind
of adverse effects on the organisms are known as xenobiotics or
toxicants and effects are called toxicities.
1 Amount of a substance in mg/kg body weight, which exerts its effect on
the organism.
Dose, of a particular substance, is an important factor in
determining whether a substance will have an adverse effect on
the organism or be eliminated from the body without any effect.
Dose also depends on the route of exposure, through which a
substance gets entry into the biological system.
Most of the xenobiotics cause an obvious alteration in
redox status of the cell or tissue by accelerating free radical
generation processes or active metabolite formation of the
xenobiotics itself. Moreover, it may alter the expression of various
genes and proteins involved in normal cell growth and
functioning. In this context a toxicological agent is capable of
interfering with the normal physiological activities in an organism
by any one or a combination of the above mentioned interactions.
In fact the generation of free radicals that is involved in oxidative
cellular injuries and alterations of gene function (either
genetically or epigenetically) are not two separable events. They
occur simultaneously. In other words, the effect a xenobiotics
exhibit in a biological system is due to a combination of genetic
and extragenetic interactions within the cell.
Inflammation is the most common response in higher
animals when there is an exposure to a toxic agent. It is simply a
protective response but persistence for a longer period (a chronic
condition) may result into a variety of diseases. Chronic
Obstructive Pulmonary Disease (COPD), pulmonary fibrosis,
Arthritis etc. are the examples of chronic2 inflammatory diseases
which may result after repeated episodes of acute inflammation
caused by a xenobiotics. Moreover, chronic inflammation is also
involved in cancer development.
2 A condition, which persists for a period of more than three months. In
most of the chronic diseases the period may be in years.
Various xenobiotics mimic the function of and interfere
with the biosynthesis of hormones which are essential for the
normal physiological functions, growth and reproduction. This
kind of xenobiotics is known as endocrine disruptors. Exposure to
such kind of chemicals is a matter of concern as they show their
effects at very low doses. Release of these chemicals into
environment is, sometime, unavoidable as some of them are used
as pesticides, in plastic industry and other chemical
manufacturing units.
Xenobiotics under the category of mutagens and
carcinogens3 are capable of altering the DNA sequences in such a
way that the cells lose their regulation of cell division and
proliferate in an uncontrolled manner. This condition can lead to
tumor formation and metastasis. These effects are lethal and a
major cause of deaths worldwide.
Alterations in neurophysiology can be responsible for
debilities in cognitive function and behavioral alterations.
Neurotoxic4 agents are capable of inducing such kind of
deleterious effects and can lead to development of
neurodegenerative diseases like Alzheimer’s and Parkinson’s.
They can affect peripheral or central nervous system and can
induce different neuropathies depending on the part affected.
These effects are mainly associated with low level of exposure.
High level of exposure to neurotoxicants can lead to coma and
sudden death.
3 All carcinogens are mutagens, but all mutagens are not carcinogens.
This is because every mutation does not result in cancer.
4 Agents which cause damages to nervous tissue and disrupts central and
peripheral nervous systems.
There is an ample generation of endogenous toxic
metabolites5, like ammonia, CO2 etc. to cause serious toxicities if
not excreted out immediately.
Excretory processes are an integral part of the body
functioning. Kidneys are the main organ involved in excretion of
not only endogenous harmful metabolites but also of the
xenobiotics and their metabolites. Lungs are mainly involved in
exhalation of CO2 and other volatile substances such as ethanol.
Liver is involved in xenobiotics metabolism and also in excretion
of certain kind of xenobiotics through bile. Injury to these organs
by a xenobiotic would hamper the excretion of endogenous
metabolites at one end, and debility in their other essential
functions at the other.
The overall scenario depicts a condition, exposure to
xenobiotics, which is not totally avoidable. The nature always tries
to make a balance between degeneration and protection6, but it
is our own activities that are responsible for the tilting of this
balance towards degeneration.
5 Normal metabolic processes release unwanted byproducts which can be
toxic to the body if not excreted out.
6 Various defensive tools exist in the body in form of detoxifying enzymes
(like Cytochrome P450, Alcohol dehydrogenase etc.), antioxidant
enzymes (Glutathione reductase, catalase, superoxide dismutase etc.)
and reducing equivalents (glutathione, NADPH, thioredoxin, glutaredoxin,
FADH etc.).
Multiple Choice Questions (MCQs)
Multiple choice questions provide a great deal of revision of a
particular subject area, whenever there is a need. The present
book contains a number of MCQs from toxicology and divided
into sets of 20 MCQs each. At the end of every set the answer
keys are given.
In addition to the answer keys, whenever it was needed a brief
explanation of the answer is also given. Full explanations of all the
positive or negative answers are out of scope of this book.
TOXICOLOGY
MCQs SET 1
MCQs Set 1
1. Which of the following case(s) can be categorized as toxicity?
A Development of chronic lung inflammation in smokers.
B Chronic rhinitis associated with influenza.
C Gout, which is inflammation of joints due to
accumulation of uric acid crystals.
D Post-surgical pus formation in a wound on forearm.
E All of the above
2. Liver is the main metabolizing organ in the body. Which of the
following statement is true in association with toxicant
metabolism and reduction of toxicity?
A Liver alone metabolizes all the toxicants and does not
need any further assistance
B Kidney plays an essential role in elimination of
metabolized toxicants through urine.
C Cardiovascular system plays an important role in
detoxification of the chemicals.
D All the chemicals are removed from the body through
bile, after metabolism in the liver.
E Gastrointestinal tract controls the flow of chemicals
into the liver.
3. Which of the following is not a natural rout of exposure to
chemicals?
A Inhalation
B Oral/Gastrointestinal tract
C Topical/Dermal
D Intraperitoneal
E None of the above
4. Which one of the following is most accurate in case of
biotransformation of the chemicals/toxicants?
A It is a synonym of the metabolism.
B Biotransformation always results in the activation of a
neutral chemical.
C Biotransformation is the transformation of one
chemical into the other irrespective of its activity.
D Biotransformation reduces toxicity by transforming
chemicals into neutral ones.
E Biotransformation does not change the chemicals, but
alters their activities in a biological system.
5. What is bioaccumulation?
A Accumulation of biological entities in a geographical
area.
B Accumulation of fossils of plants and animals
underneath several layers of mud, rock and the sand.
C Accumulation of fluid in a body part after a toxic
exposure.
D Accumulation of chemicals in body mainly in fat tissue
and bones.
E Cumulative effect of several toxicants on biological
system.
6. Bioactivation encompasses the following.
A A less harmful substance is converted into more
harmful one.
B It is a process which activates the metabolizing
enzymes in the liver.
C Bioactivation is a part of biotransformation process.
D B & C
E A & C
7. Which of the following enzymes play crucial role in detoxification
of the chemicals?
A Digestive enzymes in the gastrointestinal tract destroy
all the chemicals entering into the body.
B Cytochrome p450 oxidases, UDP-
glucuronyltransferases and glutathione S-transferases
C Superoxide dismutase, catalase, glutathione
peroxidase
D B & C
E All of the above
8. Chemicals have specific targets in the body.
A True
B Few does have targets, others are nonspecific
C False
D Depends on the route of exposure
E Chemicals decide after entering into the body.
9. Which is an acute event of toxicity?
A Lung inflammation after two days of inhalation of
metal dust.
B Liver toxicity after three daily doses of 325 mg of
paracetamol for one week only.
C Liver toxicity of three doses of 500 mg of paracetamol
for one day only.
D A toxic event occurring suddenly without an unknown
cause.
E A & C
10. Which is true regarding chronic exposure?
A Repeated exposure occurring for a period of three
months.
B Repeated exposure occurring for a period of one
month.
C Repeated exposure occurring for a period of more than
three months.
D Repeated exposure occurring for a period of one day.
E It is not categorized as chronic if no toxicity occurs.
11. Following phases are included in biotransformation of the
chemicals.
A Phase 0, Phase I, Phase II
B Phase I, Phase II and sometime Phase III
C Phase 0, Phase I, Phase II, Phase III and Phase IV
D Phase 0, Phase I,
E Phase 0, Phase I, Phase II, Phase III
12. Chemicals are toxic at any dose level.
A A few chemicals, which are lethal to humans, can be
defined like that.
B No, dose determines whether they are toxic.
C There is no dose response relationship when we talk
about highly toxic chemicals.
D The statement can be applied to inhalant toxicants,
because of lungs’ high sensitivity.
E This is true for all kind of pesticides.
13. Which of the following will have the most damaging effects?
A A hydrophilic chemical via inhalation route.
B A lipophilic chemical via dermal route.
C A lipophilic chemical via inhalation route.
D A hydrophilic chemical via oral route.
E A hydrophilic chemical via dermal route.
14. Which of the following will have the least damaging effects?
A A hydrophilic chemical via inhalation route.
B A lipophilic chemical via dermal route.
C A lipophilic chemical via inhalation route.
D A hydrophilic chemical via oral route.
E A hydrophilic chemical via dermal route.
15. Most of the chemicals are excreted out by the kidneys through
urine. Which are the other organs or modes involved in excretion
of chemicals?
A Lungs through expiration of volatile compounds.
B Sweat glands excrete a number of metabolites out of
the body.
C Through bile.
D Mammary glands
E All of the above but A, B and C are important
16. What is the biological half-life (t1/2) of a chemical?
A A chemical reduces the life of an individual to half of
the average life.
B The time in which systemic concentration of a
chemical reduces to half of its initial concentration.
C Half of the time taken to reach the maximum
concentration of chemical in the system.
D Half of the dose of a chemical which can affect the
quality of life.
E None of the above
17. Which information can be obtained from an acute toxicity study?
A Median toxic dose (TD50)
B Median lethal dose (LD50)
C No Observed Adverse Effect Level (NOEL)
D Target organ
E All of the above
18. A particular dose of a substance X is minimal toxic to animal.
Substance Y is also minimal toxic to the animals at the same dose,
but when both the substances are administered together they
show the toxicity several orders of magnitude higher than
compared with individual administrations. This is an example of:
A Potentiation
B Synergism
C Additivity
D Acute Toxicity
E Agonism
19. 1 ppm is equivalent to:
A 1 g/kg
B 1 μg/kg
C 1 mg/kg
D 1 mg/100g
E None of the above
20. Which is true about the LD50?
A Dose of a substance to which 50% of animals do not
show any response.
B Dose of a substance which kills 50% of animals
exposed.
C 50% of the dose of a substance which can kill an
animal.
D Dose of a substance which can kill 50 animals.
E 50 mg/kg dose of a substance to test the toxic
responses in animals.
Answer keys MCQs Set 1:
1. A
2. B
3. D – Intraperitoneal route of administration is used in various
experimental and clinical administrations where a test substance/drug is
delivered into the peritoneal cavity with the help of a syringe and
needle.
4. C
5. D
6. E
7. B
8. B
9. E
10. C
11. B
12. B
13. C
14. E
15. E – Theoretically toxicants/metabolites can be eliminated along with
any secretion from the body.
16. B
17. E – In an acute toxicity study any end point parameter such as death,
brain damage, liver damage etc. can be used to obtain information
regarding a particular toxicant.
18. B – In potentiation a non-toxic substance increases the toxicity of a
relatively less toxic substance. In synergism a substance shows unusually
high toxicity in presence of a less toxic substance, it is much greater than
additive effect. In agonism a substance (agonist) mimics the mechanism
of action of other substance, its opposite is known as antagonism.
19. C – 1 mg is one millionth part of a kilogram.
20. B
TOXICOLOGY
MCQs SET 2
MCQs Set 2
1 Acceptable daily intake (ADI)
A An estimate of amount of food taken daily.
B Amount of nutrients required on daily basis.
C A & B
D An estimate of the amount of substance in the food
that can be ingested daily over a lifetime by humans
without appreciable health risk.
E All of the above
2 Descriptive toxicology is concerned directly with
A Description of toxicology related research fields.
B Origin of toxicants in nature.
C Toxicity testing, which provides information for safety
evaluation and regulatory requirements?
D Describing the toxicology as a science and art to
laymen and unrelated scientific fields.
E A and D
3 The term ‘toxin’ generally refers to toxic substances that are _____
A Any kind of poisons.
B Produced by biological systems such as plants, animals,
fungi, or bacteria.
C Toxicants released as industrial effluents.
D Toxic elements of inorganic origin such as mercury,
lead, arsenic etc.
E All substances except gases.
4 The term ‘toxicant’ is used in speaking of toxic substances that are
A Specifically toxic to humans.
B Nonlethal in their effects.
C produced by organisms.
D produced by anthropogenic activities.
E Toxic to organisms such as plants, animals, fungi or
bacteria.
5 What is chemical allergy?
A The allergy described in terms of chemistry.
B An immunologically mediated adverse reaction to a
chemical resulting from previous sensitization to that
chemical or to a structurally similar one.
C When an individual exhibit adverse immunological
response against all chemicals.
D Immunological response where chemicals mediate the
process instead of antibodies.
E A and D
6 Chemical idiosyncrasy
A Extreme sensitivity to low doses or extreme
insensitivity to high doses of the chemical.
B Attribute of chemicals where they can elicit low or high
response that solely depends on their chemistry.
C Abnormal responses of individuals towards chemicals
which is determined by genetic constitution.
D Abnormal responses of individuals towards chemicals
which is determined by environmental factors.
E A and C
7 Threshold dose
A The dose level at which mortality starts occurring.
B A threshold for lethal dose.
C The dose below which no effects appear.
D A and B
E The dose at which 50% animals exhibit toxicity signs
8 No observed adverse effect level (NOAEL)
A The condition of experimental animals when no
adverse effect can be observed.
B The maximum dose at which the toxicant show no
signs of toxicity.
C A condition where adverse effects exist, but are hard
to observe.
D A and C
E The maximum dose of a chemical which caused the
death, hence no other adverse effect got observed.
9 Which of the following can be a source of toxicants?
A Car
B Fruits
C Cosmetics
D Medicines
E All of the above
10 Which one of the following elements is required by our body for
normal functioning of some enzymes?
A Mercury (Hg)
B Zinc (Zn)
C Lead (Pb)
D Antimony (Sb)
E Scandium (Sc)
11 Which one of the elements is not required by our body?
A Selenium (Se)
B Potassium (K)
C Copper (Cu)
D Arsenic (As)
E Phosphorus (P)
12 What is a reference dose (RfD)?
A Dose of a standard chemical to test the toxic doses of
other chemicals.
B An estimate of exposure to an agent for a long period
without any appreciable risk to life.
C Dose of an agent during a previous exposure to
estimate the risks of a latest exposure to the same
agent.
D All of the above
E None of the above
13 Where in the body do toxicants/chemicals get stored?
A Plasma proteins
B Body fat
C Liver and kidneys
D Bones
E All of the above
14 The mode of excretion of xenobiotics from the body. Select which
applies.
A Urinary
B Fecal (nonabsorbed, biliary excretion)
C Exhalation
D Sweat, saliva and milk
E All of the above
15 Egg shell thinning is caused by the pesticides DDT and DDE. The
mode of exposure in this case is the following:
A Direct exposure of birds to DDT and DDE.
B Nesting of birds in DDT and DDE exposed agricultural
fields.
C Movement of DDT and DDE in food chain.
D DDT and DDE don’t cause egg shell thinning.
E Direct exposure of eggs to DDT and DDE.
16 Which one of the following does not elicit toxicological interests?
A Forest fire
B Volcanic eruption
C Earthquake
D Acid rain
E Sand storm
17 What does IDLH stands for?
A International Directory For Long Term Health Effects
B Immediately Dangerous To Lung Or Heart
C International Directory For Lung And Heart Diseases
D Immediate Or Delayed Effects On Lung Or Heart
E Immediately Dangerous To Life Or Health
18 Which one of the following are the main targets of lead toxicity?
A Liver and kidneys
B Nervous system and hematopoietic system
C Heart and lung
D Bones and muscles
E Skin and hair
19 Organic mercury targets_________ whereas inorganic mercury
primarily targets__________.
A Bones, ligaments
B Liver, hematopoietic system
C Nervous system, kidneys
D Hematopoietic system, nervous system
E Liver, nervous system
20 ‘Itai-itai’ disease is caused by______
A Cadmium
B Mercury
C Lead
D Copper
E Chromium
Answer keys MCQs Set 2:
1. D
2. C
3. B
4. D
5. B
6. E
7. C
8. B
9. E
10. B
11. D – Arsenic is one of the most toxic substances and people are
exposed to it through contaminated water and soil.
12. B
13. E – Storage of toxicants within the body depends on their affinity
towards the different kinds of tissues and their components. For
example lipophilic substances are most likely to be stored in fat tissue.
14. E
15. C
16. C
17. E – represents the levels of a particular hazardous substance.
18. B – Lead targets developing nervous system and shows severe
effects in children. In adults mainly hematopoietic system is targeted.
19. C
20. A – Itai itai is a cadmium poisoning disaster occurred around 1912 in
Japan. The disease affected kidneys and bones causing severe pain in
joints and spine, hence the name itai itai (“Ouch, Ouch” or “It hurts, it
hurts”).
TOXICOLOGY
MCQs SET 3
MCQs Set 3
1 ‘Black foot’ disease is caused by ________
A Chromium
B Mercury
C Arsenic
D Lead
E Copper
2 Which of the following is not a pesticide?
A Chloropicrin
B Fluoroacetamide
C Malachite green
D Nicotine
E Malathion
3 Which of the following toxins is not produced by bacteria?
A Cholera toxin
B Aflatoxin
C Botulinum toxin
D Tetanus toxin
E Diphtheria toxin
4 Ergotism is associated with toxin produced by __________
A Bacteria
B Plant
C Fungus
D Virus
E Mycoplasma
5 Aspergillus species produce aflatoxins. Aflatoxin B1, one of the
aflatoxins, causes following toxic effect/s in humans:
A Severe liver toxicity and Carcinogenesis
B It is less toxic due to its natural origin
C Severe renal effects resulting in kidney failure
D Cardiotoxicity
E Respiratory collapse
6 Tetrodotoxin, a deadly toxin, is found in _________
A Rattle snake
B Shellfish
C Mussels
D Puffer fish
E Scorpion
7 Which is the most likely toxic effect of cosmetics?
A Skin corrosion
B Allergic contact dermatitis
C Local muscular degeneration
D Local neuronal damage
E Major risk of systemic toxicity after absorption of
chemicals through skin.
8 Gastrointestinal tract does not have any profound effect on nature
of ingested chemicals
A True
B False
C Maybe
D Gut-microflora plays important role in
biotransformation of ingested chemicals
E It does not matter whether gastrointestinal tract have
any effects on chemicals
9 Which of the following is the main way of transportation of a lipid
soluble toxicant within body?
A Filtration
B Special transport
C Endocytosis
D A and C
E Passive diffusion
10 What is the most common toxicity target of ethanol (beverage
alcohol) in humans?
A Brain
B Fetus
C Liver
D Kidneys
E Heart
11 What are the common targets of ethanol toxicity in humans?
A Liver, brain, heart and kidneys
B Liver, lungs and intestine
C Liver, brain and fetus
D Spleen, liver, brain and thymus
E All of the above
12 Methanol consumption is associated with permanent blindness.
Following comment is true regarding methanol toxicity.
A Methanol, unlike ethanol, causes direct toxicity to
optical nerves after absorption.
B Apart from the blindness, methanol is less toxic than
ethanol.
C Alcohol dehydrogenase converts methanol into
formaldehyde which causes blindness.
D Aldehyde dehydrogenase converts methanol into
formaldehyde which causes blindness.
E Alcohol dehydrogenase and aldehyde dehydrogenase
converts methanol into formic acid which causes
blindness.
13 Which part of the body is primarily affected by caffeine
intoxication?
A Heart
B Brain
C Liver
D Kidneys
E Gastrointestinal tract
14 Leaves from the following plants are the significant source of
caffeine.
A Coffea arabica
B Erythroxylum coca
C Cola acuminata
D Camellia sinensis
E Theobroma cacao
15 Who first of all described the association between chemical
exposure and cancer?
A Percival Pott
B Paracelsus
C Mathieu Orfila
D Bernardino Ramazzini
E Oswald Schmiedeberg
16 Who is the father of forensic toxicology?
A Louis Lewin
B Paracelsus
C Mathieu Orfila
D Bernardino Ramazzini
E Oswald Schmiedeberg
17 Who is the father of occupational medicine?
A Louis Lewin
B Paracelsus
C Mathieu Orfila
D Bernardino Ramazzini
E Oswald Schmiedeberg
18 Exposure to __________ is associated with occupation.
A Aflatoxins
B Cigarette smoke
C Formaldehyde
D Ethanol
E Acetaminophen
19 Exposure to __________ is associated with lifestyle.
A Chromium
B Benzene
C Benzidine
D Nicotine
E Asbestos
20 Mesothelioma (cancer of lining covering internal organs) is
associated with exposure to ____
A Benzene
B Nickel
C Mercury
D Arsenic
E Asbestos
Answer keys MCQs Set 3:
1. C
2. C - Malachite green is used as a dye in leather and paper industries.
3. B
4. C
5. A
6. D – Liver, intestine and skin of puffer fish is known to be source of
tetrodotoxin, a potent neurotoxin, which is produced by symbiotic
bacteria Pseudoalteromonas tetraodonis and species of Vibrio and
Pseudomonas.
7. B
8. D
9. E
10. C
11. C – After liver and brain, the ethanol causes fetal toxicity known as
Alcohol Fetal Syndrome (AFS) causing severe mental and physical
damages in developing babies.
12. E
13. B
14. D
15. A
16. C
17. D
18. C – Formaldehyde is used in shoe industry and has been associated
with nasal cancer.
19. D
20. E
TOXICOLOGY
MCQs SET 4
MCQs Set 4
1 Benzene is known to cause _________ cancer.
A Bone marrow
B Cervical
C Ovarian
D Prostate
E Breast
2 Thalidomide is known for its ___________ effects.
A Teratogenic
B Carcinogenic
C Mutagenic
D Neurotoxic
E Hepatotoxic
3 The very well reported birth defect of thalidomide is________
A Cleft palate
B Congenital heart disease
C Ectrodactyly
D Club foot
E Phocomelia
4 What is the expression of dose of a substance?
A mg/lbs
B g/kg
C μg/100g
D μg/kg
E mg/kg
5 Ppm (parts per million) is equivalent to _________
A ml/L
B mg/kg
C μg/kg
D mg/L
E μg/g
6 Which of the following pesticides is acetylcholinesterase inhibitor?
A Formamidines
B DDT and DDE
C Nicotine
D Pyrethroids
E Organophosphates
7 Which one is an antidote for organophosphate poisoning?
A Activated charcoal
B EDTA
C Ipecac
D Atropine
E BAL
8 Dose is defined as the __________________________
A Amount of substance which is released into the
environment.
B Amount of substance which reaches the target site in
the body.
C Amount of substance which is converted into active
metabolite to exert toxic effect.
D Amount of substance which enters into the body.
E Amount of substance is bound to the plasma proteins.
9 Which one of the following is true regarding Dose-response
relationship?
A Response is directly proportional to the dose of a
substance.
B Dose exhibits ‘all or none’ response.
C The dose which elicit a toxic response.
D The dose which does not induce any response.
E None of the above.
10 What is a quantal dose-response?
A Continuous responses in an individual against varying
doses of a substance.
B The responses which can be quantified.
C The most probable responses in experimental animals
after toxicant exposure.
D The responses which cannot be quantified.
E Individual responses distributed in a population to
different doses of a substance.
11 Following is an example of quantal dose-response observation.
A Toxic effects in response to increasing doses of a
chemical.
B Effect of a dietary supplement on body weight in a
population.
C Observation of median lethal dose (LD50) in rats.
D Death of an individual after a snake bite.
E B and C
12 What is a graded dose-response?
A Continuous responses in an individual against varying
doses of a substance.
B Responses to a single exposure at different time
interval.
C Effects other than the major one against a dose of a
substance.
D Individual responses distributed in a population to
different doses of a substance.
E A and C
13 What is the antidote for methanol poisoning?
A Atropine
B British anti lewisite
C Charcoal
D Ethanol
E Disodium EDTA
14 Which one of the following is not an air pollutant?
A Ozone
B Argon
C Nitrogen oxides
D Sulfur oxides
E Hydrogen sulfide
15 What is the mode of action of carbon monoxide (CO)?
A It reduces the oxygen absorption by directly or
indirectly damaging the alveoli.
B It destroys the red blood cells.
C Combines with hemoglobin to reduce oxygen carrying
capacity of RBCs.
D It has a corrosive effect on airways, specially bronchi
and bronchioles.
E It alters the structure of the hemoglobin.
16 Sulfur dioxide (SO2) affects the respiratory system by _________
A Forming sulfurous acid in respiratory tract, resulting in
irritation.
B Reducing oxygen carrying capacity of blood.
C Causing allergy
D Carcinogenesis
E Respiratory collapse
17 Which of the following participate in formation of photochemical
smog?
A Oxides of sulfur
B Hydrogen
C Ozone
D Oxides of nitrogen
E Carbon dioxide
18 What is the cause of acid rain?
A Over production of acids in factories.
B Increased environmental concentration of Oxides of
nitrogen and sulfur due to air pollution.
C Increased use of acid in household cleaning.
D Increased aviation activities.
E Carbon dioxide (CO2) and global warming.
19 Which of the following is not a possible route of exposure to a
toxicant in the environment?
A Inhalation
B Ingestion
C Dermal absorption
D Transdermal
E None of the above
20 What happens to DDT when it enters the body?
A It is stored in the bones.
B It is water soluble and easily excreted out from the
body.
C It is fat soluble and stored in fat tissue.
D It is converted into an active metabolite.
E It bypasses the metabolism and excreted as such.
Answer keys MCQs Set 4:
1. A
2. A
3. E – Phocomelia, a condition where long limb bones are not formed in
fetuses. Arms without long bones appear like flippers hence the
condition is called ‘Flipper Arms’ a common condition caused by
thalidomide.
4. E – Dose of a substance is always measured in mg/kg body weight.
Where the measurement in case of toxic gases or fumes the exposure
level is measured as concentration in ppm/m3.
5. B
6. E
7. D
8. D
9. A
10. E
11. E
12. A
13. D – Ethanol is agonist for methanol and competes with it for
metabolism by alcohol dehydrogenase and aldehyde dehydrogenase,
thus minimizes the formation of toxic metabolites of methanol,
formaldehyde and formic acid.
Alcohol Dehydrogenase Aldehyde Dehydrogenase
Methanol ---------------> Formaldehyde ------------------> Formic acid
(Causes blindness)
Alcohol Dehydrogenase Aldehyde Dehydrogenase
Ethanol ----------------> Acetaldehyde -------------------> Acetic acid
14. B
15. C – Carbon monoxide has around 200 times greater affinity for
hemoglobin than oxygen.
16. A – SO2 reacts with aqueous layer on lung epithelium to form
sulfurous acid.
SO2 + H2O → H2SO3
17. D
18. B - Oxides of nitrogen and sulfur reacts with environmental moisture
to form nitric acid and sulfuric acid respectively.
19. D – Transdermal route is used to deliver certain drugs in clinical set
up.
20. C
TOXICOLOGY
MCQs SET 5
MCQs Set 5
1 Who is most likely to exhibit toxic effects of environmental
tobacco smoke (ETS).
A A person with asthma
B An obese teenager
C An athlete
D A child in 2nd grade
E A driver who smokes
2 Which of the following pneumocyte clears the particles deposited
in the lungs?
A Type I pneumocyte
B Type II pneumocyte
C Clara cells
D Fibroblasts
E Alveolar macrophages
3 Emphysema is an inflammatory tissue damage, which can be
caused by toxicants. Which organ is affected in case of
emphysema?
A Kidneys
B Heart
C Lung
D Skin
E Nervous system
4 Which protein is mainly damaged in emphysema?
A Collagen
B Elastin
C Keratin
D Albumin
E globulin
5 Cigarette smoking is the main cause of Chronic Obstructive
Pulmonary Disease, which is a combination of following ailments.
A Emphysema, chronic bronchitis, asthma
B Chronic bronchitis, bronchiolitis, edema
C Asthma, tracheitis, necrosis
D Emphysema, cough, sneezing
E Allergic cough, rhinitis, lower respiratory infections
6 Which of the following statements is true?
A Light cigarettes are safer than the regular ones.
B Electronic cigarettes are less harmful to the lungs and
heart.
C Filtered cigarettes are way safer than the non-filtered
cigarettes.
D Herbal cigarettes are natural hence harmless.
E None of the above
7 What does PM10 indicate?
A Stage of a lung disease.
B Size of inhalant particles enough to reach and be
deposited in the alveoli.
C Number of cigarettes per day to cause lung cancer in
10 years.
D Measure of pollutants in a unit volume of inhalant air.
E Degree of concentration of toxicants present in the air.
8 A plenty of free radicals and oxidants are released with cigarette
smoke. What does these oxidative species actually do to the
lungs?
A They alter the secretion of surfactants which protect
the lung epithelium.
B They block clearance of particulate matter by inhibiting
cilia movement.
C They damage hemoglobin and reduce oxygen carrying
capacity of the blood.
D They can damage cellular macromolecules like DNA,
proteins and lipids and exert severe cellular damages
leading to several diseases.
E A and B
9 Four of the following are the sources of hazard and one is a
hazard. Which one is a hazard?
A Asbestos mining
B Chemical factory
C Cigarette
D Metal fumes
E Automobiles
10 Which of the following is not helpful in determining the dose of a
toxic substance?
A Body weight
B Animal species (including human)
C Origin of toxicant
D Chemico-biological nature of the toxicant
E Body surface area
11 Following are the normal functions of the metals in the body
except _________.
A Calcium in bone formation
B Phosphorus in ATP
C Iron in hemoglobin
D Lithium in sodium channels activities
E Magnesium in enzyme functions
12 What are the effects of cellular hypoxia in case of carbon
monoxide poisoning?
A Alveolar collapse
B Lung edema
C Brain damage and cardiac dysrhythmia
D Disrupted glycolysis
E Disrupted hematopoiesis
13 Why activated charcoal is administered in case of oral poisoning?
A It deactivates toxicants
B It increases the metabolic processes of detoxification
C It chemically detoxifies the toxicants
D It chelates metal circulating in the blood
E It reduces absorption of toxicants by adsorbing them
14 Which of the following is associated with acetaminophen toxicity?
A Methanol
B N-acetyl-p-benzoquinone imine
C 3-(1-methylpyrrolidin-2-yl)pyridine
D Benzo(a)pyrene
E Diethyl nitrosamine
15 Who is the most susceptible to hepatotoxicity by high doses of
acetaminophen (paracetamol)?
A A child in second grade
B An old lady
C A person on fasting
D An alcoholic
E A factory worker
16 What kind of toxicity does ammonia cause?
A Cardiotoxicity
B Tissue corrosion
C Neurotoxicity
D Reduced gastric motility
E hepatotoxicity
17 Which one of the arsenic compounds causes hemolysis?
A Arsenic trioxide (As2O3)
B Arsenic pentoxide (As2O5)
C Arsenic pentafluoride (AsF5)
D Arsenic trisulfide (As2S3)
E Arsine (AsH3)
18 Asbestos is a group of naturally occurring silicates, which include
____________.
A Chrysotile and anthophyllite
B Amosite and actinolite
C Crocidolite
D Tremolite
E All of the above
19 Sodium azide (NaN3) which is used in car air bags and as
preservative in laboratories has a mechanism of toxicity similar to
_________.
A Aconite
B Cyanide
C Nicotine
D Arsenic
E Lithium
20 Botulinum toxin causes death by___________.
A CNS depression and coma
B Cardiac failure
C Respiratory failure
D Severe hemolysis
E Renal failure
Answer keys MCQs Set 5:
1. A – Asthma can be triggered by tobacco smoke and pose severe
damage. Children are also prone to toxic insults caused by tobacco
smoke. However, tobacco smoke is toxic for anyone exposed.
2. E
3. C – In emphysema lung alveoli are damaged, become enlarged and
loose flexibility. It is one of the several conditions collectively known as
chronic obstructive pulmonary disease (COPD).
4. B
5. A
6. E
7. B
8. D – Oxidants exert their effect by damaging cellular macromolecules.
9. D
10. C
11. D – Lithium is used as medicine in mental illnesses like depression,
schizophrenia and bipolar disorder.
12. C
13. E
14. B - N-acetyl-p-benzoquinone imine is reactive metabolite of
acetaminophen which is responsible for liver damage and other
associated toxic effects.
CYP 2E1 (Liver damage)
Acetaminophen ----------------> N-acetyl-p-benzoquinone imine (NAPQI)
(Glutathione)
---------------> NAPQI-glutathione conjugates ---------> Excretion
15. D
16. B
17. E
18. E
19. B
20. C – Botulinum causes paralysis of respiratory muscles.
TOXICOLOGY
MCQs SET 6
MCQs Set 6
1 Cadmium is a highly toxic metal that causes__________.
A Chemical pneumonitis
B Damage to renal tubules
C Cancer
D GI tract irritation
E All of the above
2 What is the main concern with the chlorinated hydrocarbon
pesticide when compared with other classes of pesticide?
A They are highly neurotoxic
B They can cause death
C They persist in the environment
D They damage the crops
E They can kill the cattle
3 Chlorine has toxic effects similar to _________.
A Arsine
B Phosgene
C Mustard gas
D Ammonia
E Carbon monoxide
4 Which one of the following is not associated with sea food
toxicity?
A Ciguatoxin
B Aflatoxin
C Tetrodotoxin
D Saxitoxin
E Okadaic acid
5 What are the most likely toxic effects of Iodine?
A Corrosive effects like that of chlorine
B Oxidative damage to cellular molecules
C Disturbed thyroid functions
D Adrenal gland disturbances
E Disturbed pancreatic gland functions
6 Ipecac syrup is derived from Cephaline ipecacuanha plant and used
for ________.
A Inducing emesis in case of oral poisoning
B Supporting cardiac function in case of pesticide toxicity
C To potentiate liver function
D lung inflammatory diseases
E GI tract cleaning
7 Which is the common target of toxicity of lithium, manganese and
magnesium?
A Cardiac muscles
B Bones
C Skeletal muscles
D Central nervous system (CNS)
E Immune system
8 Common toxicity target of elemental mercury and methyl mercury
is ________.
A Kidneys
B Liver
C Muscles
D Skin
E CNS
9 Common toxicity target of inorganic mercuric salts and organic
mercury is ________.
A Kidneys
B Liver
C Muscles
D Skin
E CNS
10 Common toxicity target of elemental mercury, inorganic mercuric
salts and organic mercury is ________.
A Kidneys
B Liver
C Muscles
D Skin
E CNS
11 Major rout of exposure to elemental mercury is _______.
A Dermal
B Inhalation
C Ingestion
D Intravenous
E Intramuscular
12 ‘Metal fume fever’ is caused by inhalation of fumes of _______.
A Zinc oxide
B Elemental mercury
C Chromium oxide
D Ferric chloride
E Any of the metal oxides
13 A 28 year old person accidentally consumed high doses of a
nitrate. What sort of toxicity is expected in that person?
A Reduced oxygen carrying capacity/hypoxia
B Acute hypotension
C Increased hypoxia and a little hypotension
D Increased hypotension and a little hypoxia
E Respiratory collapse
14 Toxicity targets of nonsteroidal anti-inflammatory drugs (NSAIDs)
include_______.
A CNS and muscles
B Musculoskeletal system and liver
C GI tract and kidneys
D Cardiovascular system
E Respiratory system
15 Morphine is obtained from the following plant
A Nicotiana tabacum
B Erythroxylum coca
C Atropa belladonna
D Papaver somniferum
E Datura alba
16 Opioids like heroin and morphine can cause death by _________.
A CNS depression
B Cardiac failure
C Respiratory depression and failure
D Renal failure
E Hemolytic anemia
17 Target organ of the herbicide paraquat is ________.
A Liver
B Kidney
C Endocrine glands
D Lung
E Reproductive organs
18 Which of the following is not recommended in case of paraquat
poisoning?
A Charcoal administration
B Oxygen administration
C Washing of exposed skin
D Maintenance of open airway
E Fluid administration
19 What are the effects, other than the systemic effects of the
smoke?
A Thermal damage to the airways
B Irritation
C Asphyxia
D A, B and C
E B and C
20 Which of the following statement is associated with first order
kinetic?
A Excretion can be increased by increasing pH
B Amount of excretion does not depend on the
concentration of the toxicant
C Amount of excretion can be increased by diuresis
D Amount of excretion depends on the concentration of
the toxicant
E Excretion is not associated with the metabolism
Answer keys MCQs Set 6:
1. E
2. C
3. D – Chlorine and ammonia, both have corrosive effects.
4. B – Aflatoxin is produced by a fugus , Aspergillus flavus, which usually
contaminate peanuts.
5. C – Iodine is essential for the production of two thyroid hormones
riiodothyronine (T3) and thyroxin (T4), which regulates the metabolism.
Toxic doses of iodine disrupt this hormone system.
6. A
7. D
8. E
9. A
10. D
11. B
12. A
13. B – Nitrates cause vasodilation and reduces blood pressure.
14. C
15. D
16. C
17. D
18. B
19. D
20. D – In first order kinetics excretion is directly proportional to the
concentration of the substance.
TOXICOLOGY
MCQs SET 7
MCQs Set 7
1 Which one belongs to zero order kinetics?
A Half-life remains constant even if the concentration of
the compound is increased
B Half-life is increased with the increasing concentration
of the compound
C Half-life is independent of the amount of compound
D Half-life is indirectly proportional to the amount of
excretion
E Half-life is directly proportional to the amount of
excretion
2 Wilson’s disease is associated with the accumulation of following
metal-
A Zinc
B Calcium
C Manganese
D Iron
E Copper
3 Which of the following favors the accumulation of the toxicants in
biological system?
A Molecular size and chemical properties
B Hydrophilicity and protein binding
C Resistance to metabolism and lipophilicity
D Reuptake in the gut and enzyme inhibition
E Entrance in the food chain and food web
4 Most of the copper in the body remains bound to __________
A Ceruloplasmin
B Plasminogen
C Albumin
D Globulin
E Collagen
5 Antidote/s administered in case of copper poisoning
A Penicillamine
B Disodium EDTA
C Dimercaprol
D A and B
E A and C
6 Antidote for the arsenic poisoning
A N-acetylcysteine
B Sodium nitrate
C Dimercaprol
D EDTA
E Fuller’s earth
7 Antidote for cyanide poisoning
A N-acetylcysteine
B Sodium nitrate
C Dimercaprol
D EDTA
E Fuller’s earth
8 Antidote for paraquat
A N-acetylcysteine
B Sodium nitrate
C Dimercaprol
D EDTA
E Fuller’s earth
9 Antidote for acetaminophen/paracetamol
A N-acetylcysteine
B Sodium nitrate
C Dimercaprol
D EDTA
E Fuller’s earth
10 Therapeutic index is expressed as __________.
A LD50/TD50
B LD50/ED50
C TD50/LD50
D TD50/ED99
E ED50/LD50
11 Margin of safety is expressed as __________.
A LD1/ED99
B LD50/ED50
C TD50/LD50
D TD50/ED99
E ED50/LD50
12 What is the ‘First pass effect’?
A When a larger part of the drug/toxicant bypasses the
liver and reaches circulation.
B When a larger part of the drug/toxicant first passes
through the circulation and then reaches the liver for
metabolism.
C When a larger part of the drug/toxicant first gets
converted into active metabolite in the circulation.
D When a larger part of the drug/toxicant passes directly
through urine without reaching to the liver.
E When a larger part of the drug/toxicant is metabolized
by GIT and/or liver before reaching to the circulation.
13 In the body most of the drugs remain bound to the plasma
proteins. Which one of the following is the preferred bond?
A Ionic bonds
B Hydrogen bonds
C Van der Waal’s forces
D Hydrophobic interactions
E All of the above
14 Xenobiotic metabolism will basically lead to _________.
A Bioaccumulation of the xenobiotic
B Altered chemical structure
C Increased toxicity
D Decreased toxicity
E Increased renal excretion
15 Which is not a part of phase I biotransformation of toxicants?
A Oxidation
B Hydrolysis
C Reduction
D Glutathione conjugation
E Epoxide hydrolase
16 Following are the reactions included in phase II biotransformation,
except _________.
A Glucuronidation
B Quinone reduction
C Sulfonation
D Acetylation
E Methylation
17 Which is the most important enzyme in xenobiotic
biotransformation?
A Epoxide hydrolase
B Carboxylesterases
C Flavin Monooxygenases
D Cytochrome b5
E Cytochrome P450
18 Alcohol dehydrogenase, an important enzyme in
biotransformation of alcohols, catalyzes the following type of
reaction-
A Reduction
B Hydrolysis
C Oxidation
D Hydroxylation
E Acetylation
19 Which one is the most common conjugation reaction in phase II
biotransformation?
A Glucuronidation
B Sulfonation
C Acetylation
D Methylation
E Glutathione conjugation
20 UDP–glucuronosyltransferases (UGTs), which catalyze the
glucuronidation reactions in phase II biotransformation, are
located in ______________ within the cell.
A Cytoplasm
B Endoplasmic reticulum
C Mitochondria
D Lysosomes
E Peroxisomes
Answer keys MCQs Set 7:
1. B – in zero order kinetics the rate of excretion remains constant, so
increasing concentration means increase in half life.
2. E
3. C
4. A
5. E
6. C
7. B
8. E - Fuller’s earth works as an adsorbent and reduces the absorption of
paraquat into the system.
9. A
10. B – lower value of therapeutic index (LD50/ED50) indicates higher risk
of toxicity of therapeutic drugs. It is also expressed as TD50/ED50.
(ED = Effective dose; LD = Lethal dose; TD = Toxic dose)
11. A
12. E
13. E
14. B - Xenobiotic metabolism basically alters the chemical structure in
order to increase the exretion of the substance which ultimately may
lead to increased or decreased toxicity.
15. D - Glutathione conjugation and other conjugation reactions are the
part of Phase II biotransformation.
16. B
17. E
18. C
19. A
20. B
TOXICOLOGY
MCQs SET 8
MCQs Set 8
1 Cytochrome P450 enzymes, which catalyze the largest portion of
phase I biotransformation, are located in ______________ within
the cell.
A Cytoplasm
B Endoplasmic reticulum
C Mitochondria
D Lysosomes
E Peroxisomes
2 Bioactivation means _________________
A Activation of biological processes by means of
drugs/toxicant exposure
B Activation of a species after a long seasonal dormancy
period
C Conversion of a less- or non-toxic compound into a
potentially toxic compound by metabolic process
D Improvement of work efficiency using energetic
nutrients
E None of the above
3 If an enzyme is microsomal, it means it is __________
A Located in peroxisomes
B Lysosomes
C Endoplasmic reticulum
D Golgi apparatus
E Mitochondria
4 Sulfotransferases (SULT) catalyzes sulfonation reactions in Phase II.
There are two major classes of these enzymes i) membrane bound
and ii) soluble respectively located in _______
A Endoplasmic reticulum and cytoplasm
B Mitochondria and extracellular matrix
C Endoplasmic reticulum and extracellular matrix
D Golgi apparatus and cytoplasm
E Mitochondria and cytoplasm
5 Sulfonation involves the transfer of the following to the xenobiotic
A SH‾
B SO4‾
C S2‾
D SO3‾
E SO42‾
6 What is the cofactor in sulfonation reactions?
A 3’-phosphoadenosine-5’-phosphosulfate (PAPS)
B Sodium dodecyl sulfate
C Adenosine triphosphate (ATP)-sulfurylase
D Magnesium sulfate
E Alkyl sulfonate
7 Which is the most common cofactor in one of the main phase II
biotransformation reactions?
A Uridine Diphosphate Glucose
B Uridine Diphosphate Glucuronic Acid
C Uridine Diphosphate Xylose
D Uridine Diphosphate Galactose
E All of the above
8 Which of the following is an example of bioactivation?
A Acetaminophen → N-acetyl-p-benzoquinine imine
B Malathion → Malaoxon
C Aflatoxin → Aflatoxin-8,9 epoxide
D Benzo(a)pyrene →benzo(a)pyrene 7,8-diol 9,10-
epoxide
E All of the above
9 Following are the examples of adverse effects of toxicants except
_____.
A Decreased lifespan
B Increased incidences of limb injuries
C Altered physiology and biochemistry
D Behavioral instability
E Decreased growth
10 ‘Ames test’ is used to detect _______.
A Salmonella typhi infection.
B Salmonella typhimurium infection.
C Mutagenic potential of chemicals using in-vitro test
with mutant strains of Salmonella typhimurium.
D Carcinogenic potential of chemicals using in-vitro test
with HeLa cancer cell lines.
E Hepatotoxic potential of chemicals in cell cultures.
11 Anthracosis is a lung condition common in _______.
A Gold miners
B Diamond miners
C People doing research on Bacillus anthracis
D Coal miners
E Wine distillers
12 Argyrosis is caused by _______
A mercury
B cadmium
C silver
D lead
E arsenic
13 Bagassosis disease is associated with the following organ.
A Heart
B Lung
C Liver
D Kidneys
E Brain
14 The target organ of beryllium toxicity is _____.
A Kidneys
B Heart
C Liver
D Lung
E Brain
15 Which of the following interface does not exist?
A Blood-Brain Barrier
B Blood-Testis Barrier
C Blood-Follicle Barrier
D Blood-Pancreas Barrier
E Blood-Placenta Barrier
16 Workers involved in cotton processing may suffer with ______.
A tuberculosis
B silicosis
C siderosis
D stannosis
E byssinosis
17 Ceiling Value (CV), a term used in occupational toxicology,
denote_______
A Ceiling of a workplace should be well ventilated.
B Concentration of an airborne toxicant in workplace.
C Lethal concentration of a toxicant in workplace area.
D level of heavy metal fumes reaching up to the ceiling.
E none of the above.
18 Burning sensation after exposure to chilli pepper is an example of
________.
A paresthesia
B sensitization
C receptor activation
D contact dermatitis
E chemesthesis
19 Chemosis is __________.
A swelling of conjunctiva due to chemical exposure.
B a lung condition after chemical inhalation.
C occupational disease in chemical factory workers.
D osmosis of chemicals.
E inflammation of limbs after chemical exposure.
20 Ciguatera toxin is produced by _______.
A Barracuda fish.
B Pseudoalteromonas tetraodonis
C Gambierdiscus toxicus
D Fugu (Puffer fish)
E Aspergillus flavus
Answer keys MCQs Set 8:
1. B
2. C
3. C – In experimental set up endoplasmic reticulum are isolated using
ultracentrifugation of the tissue homogenates. The endoplasmic
reticulum cannot be collected in its natural form they are collected as
small globular membrane bound forms known as microsomes, hence the
enzymes found in endoplasmic reticulum are called microsomal
enzymes.
4. D
5. D
6. A
7. B
8. E
9. B
10. C
11. D
12. C – greyish or dark grey pigmentation due to accumulation of silver
in tissues.
13. B – Bagassosis is caused by inhalation of constituents of sugar cane.
14. D
15. D
16. E
17. B
18. E
19. A – It result in swelled surroundings of the eyes.
20. C – Barracuda fishes are found contaminated with ciguatera toxin.
TOXICOLOGY
MCQs SET 9
MCQs Set 9
1 In ‘comet assay’ we measure the ________.
A extent of hepatotoxicity.
B extent of damage in spleen.
C extent of damage in renal tubules.
D extent of DNA damage in single cells.
E pancreatic damage.
2 What is ‘bioavailability’?
A Availability of biological entities for exposure to
toxicants.
B Ability of chemicals to be absorbed by organisms.
C Chemicals in an approachable vicinity.
D Availability of biologically produced agents.
E None of the above.
3 Tooth enamel malformation is caused by _______ toxicity.
A copper
B phosphorus
C fluoride
D iron
E lithium
4 Who discovered the carcinogenic effects of the chemicals?
A Percivall Pott
B Paracelsus
C Orfilla
D Bernardino Ramazzini
E Socrates
5 Endotoxins are _________.
A produced by yeasts
B fungal toxin
C bacterial toxins
D produced by mycoplasmas
E viral toxins
6 Endotoxins are released after death of some microorganisms.
A True
B False
7 ‘Environmental medicine’ encompasses the management of
damages caused by toxicants to the environment.
A True
B False
8 In ‘first order kinetics’ reaction rate is indirectly proportional to
the concentration of the substance.
A True
B False
9 ‘First pass effect’ is the biotransformation of chemicals in the
intestine before reaching to the circulation.
A True
B False
10 Genotoxicity involves ________.
A alteration in DNA sequence.
B toxic damages to the DNA, such as DNA strand breaks.
C Chromosomal abnormalities.
D A & B
E All of the above
11 Chemical carcinogenesis is not a toxic effect.
A True
B False
12 ‘Guinea-pig maximization test’
A used to asses skin carcinogens.
B used to test maximum damage a toxicant can cause.
C used to screen contact allergens.
D used to assess penetration potential of cosmetics.
E an alternative to LD50 experiment.
13 ‘Half-life’ and ‘Half time’ are different parameters.
A True
B False
14 ‘Hazard’ indicates inherent toxic properties of a substance, in
other words a ‘source of danger’.
A True
B False
15 ‘Hormesis’ is a phenomenon where generally toxic/non-essential
substances show beneficial effects at low doses.
A True
B False
C maybe
16 An adverse condition is called ‘iatrogenic’ when it arises from
lack of medical treatment.
A True
B False
17 When an individual show unusually high sensitivity towards a
substance, the condition is known as _________.
A Idiopathic
B Icterus
C Anaphylaxis
D Idiosyncrasy
E Hypersensitivity
18 ‘Micronucleus’ formation is an example of ________.
A teratogenicity
B carcinogenicity
C genotoxicity
D cytotoxicity
E B & C
19 Apoptotic tissue injuries are accompanied by inflammation.
A True
B False
20 Substances in nanoforms (Nanosized Particles [NSPs], <100 nm)
are less toxic than their crude/normal forms.
A True
B False
Answer keys MCQs Set 9:
1. D
2. B
3. C
4. A
5. C – Endotoxins are integral part of cell walls of certain Gram-negative
bacteria. They are also known as lipopolysaccharides (LPS).
6. A
7. B
8. B
9. B
10. E
11. B
12. C
13. B
14. A
15. A
16. B - An adverse condition is called ‘iatrogenic’ when it arises from
medical treatment itself.
17. D
18. C
19. B – Necrotic injuries induce inflammation.
20. B
TOXICOLOGY
MCQs SET 10
MCQs Set 10
1 Oxidative stress is a condition where reactive oxygen species
(ROSs) generation outweighs the body’s capacity to counter them.
A True
B False
2 Which of the following statements belongs to ‘toxicodynamics’?
A Mercury vapors inhaled and accumulated in the CNS.
B Binding of copper with ceruloplasmin.
C Elimination of lead after chelation with dimercaprol.
D Inhibition of ALAD enzyme in case of lead toxicity.
E Urine levels of NAPQI-glutathione level.
3 A toxicant produced by an animal is called _________.
A toxin
B biotoxin
C venom
D poison
E mycotoxin
4 An obese person survives a long period of exposure to a lipophilic
toxicant T. After a certain period he pass through a period of
starvation. What are the prospects in terms of toxicology?
A He survived toxicant exposure, he will survive through
starvation period as well.
B He will be in good shape after the period is over.
C Chances are there that he suffer severe toxicity by
toxicant T which should be released from the fat tissue
decomposing during starvation.
D He has a lot of fat, so the starvation will not affect him.
E None of the above.
5 A substance ‘S’ follows zero order kinetics. A person is exposed to
a certain amount ‘A’ of ‘S’ on daily basis. The half-life of amount
‘A’ of ‘S’ is one day. The person was exposed to ‘S’ for three days.
How much should he wait for complete elimination of the ‘S’?
A 6 days
B 5 days
C 4 days
D 3 days
E 9 days
6 In Ames test a substance ‘M’ demonstrates significant mutagenic
potential. What implications can be drawn from this information?
A The substance ‘M’ is surly a carcinogen.
B ‘M’ will induce cancer in animal models but not in
humans.
C ‘M’ may or may not have carcinogenic potential.
D ‘M’ is an environmental contaminant.
E Substance ‘M’ can be used as disinfectant in industrial
and household set ups.
7 Gene mutations caused by chemicals initiates the process of
carcinogenesis. Which kinds of genes are mutated in this
process?
A Oncogenes
B Tumor suppressor genes
C Protooncogenes
D All of the above
E B & C
8 Genotoxicity always leads to carcinogenesis/cancer.
A True
B False
9 DNA damage generally results in __________.
A Cancer formation
B Unregulated cell proliferation
C Teratogenicity
D Reproductive system damage
E Cytotoxicity
10 Different agents of chemical, biological or physical origin have
potential to alter DNA sequence and capable of causing
cancer. What is your opinion regarding the above statement?
A Only chemicals can cause cancer.
B No physical agent is known to cause cancer.
C Studies show that the agents of chemical, biological
or physical origin can cause cancer.
D The statement is not true.
E Viruses do not belong to any category still they can
cause certain types of cancer.
11 “All substances are poison….”. Under the aegis of this statement,
when do you see the toxicity of table sugar?
A In case of obesity and related heart problems.
B Generation of extra heat and related discomfort during
summers.
C Dental caries.
D Degeneration of different tissues/organs in case of
diabetes mellitus.
E Sugar is not a poison.
12 Use of performance enhancing drugs is illegal, but the drugs itself
are not toxic to health at all.
A True
B False
13 Assume all the types of Ultra Violet (UV) rays (A, B and C)
reaches down to the earth. Which of them all will be most damaging to
the life?
A UV C
B UV A
C UV B
D UV A and C are equally damaging
E UV A and B are equally damaging
14 Skin cancer is one of the major toxic effects of the UV exposure.
What is/are the other major health effect/s?
A Osteoporosis
B Alopecia
C Cataract
D Hyperpigmentation
E Contact dermatitis
15 Which one of the following is not an ionizing radiation?
A X rays
B infrared rays
C Gamma rays
D UV rays
E Alpha rays
16 Chemical carcinogenesis has following stages (select all that
applies).
A Initiation
B Immigration
C Promotion
D Metastasis
E Progression
17 Which of the following is a non-solid tumor?
A Mesothelioma
B Renal cell carcinoma
C leukemia
D Basal cell carcinoma
E Breast cancer
18 What is the common mechanism of action of most of the plant
based products?
A Hormesis
B Antioxidant activity
C Oxidative stress induction
D Molecular mechanisms
E B & D
19 Alternative medicine includes only herbal medicine.
A True
B False
20 Herbal formulations are always safe and do not need any
toxicological evaluation and regulation.
A False
B True
Answer keys MCQs Set 10:
1. A
2. D – Other statements are associated with ‘toxicokinetics’ where
movement of toxicants is observed in order to study what body does to
the toxicants.
3. C
4. C
5. D
6. C
7. E
8. B
9. E
10. C
11. D
12. B
13. A – UV C has shortest wave length and highest energy level in UV
group and, UV B comes second and the UV A is third in the order.
However, in natural conditions only UV A reaches to the ground in
significant amount, which is damaging to the life. Ozone layer filters out
UV C completely and UV B partially.
14. C
15. B
16. A,C,D,E
17. C
18. E
19. B
20. A
Bibliography
Bibliography
• A small Dose of Toxicology (Second Edition). Steven G. Gilbert.
Healthy World Press, US, 2012.
• A text book of modern toxicology (Third Edition). Ernest Hodgson
(Ed) John Wiley & Sons, Inc., Hoboken, New Jersey, 2004.
• Casarett and Doull’s Toxicology: The Basic Science of Poisons
(Seventh Edition). Curtis D. Klaassen (Ed). McGraw-Hill, New York,
2008.
• Lu’s Basic Toxicology: Fundamentals, Target Organs, and Risk
Assessment (Fifth Edition). Lu FC, and Kacew S. Informa Healthcare
USA, Inc., New York, 2009.
• Principles Of Toxicology: Environmental and Industrial Applications
(Second Edition). Williams PL, James RC, Roberts SM (Eds). John
Wiley & Sons, New York, 2000.
• Glossary Of Terms Used In Toxicology, 2nd Edition (IUPAC
Recommendations 2007), International Union Of Pure And Applied
Chemistry, Chemistry And Human Health Division. Duffus HS,
Nordberg M and Templeton DM. Pure and Applied Chemistry,
Vol.79, No.7, pp.1153–1344, 2007.
View
publication
stats
View
publication
stats

More Related Content

Similar to Toxicology

Chapt08 Lecture
Chapt08 LectureChapt08 Lecture
Chapt08 Lecturerpieper
 
A current genetic and epigenetic view on human
A current genetic and epigenetic view on humanA current genetic and epigenetic view on human
A current genetic and epigenetic view on humanmariannajd
 
pathophysiology and psychodynamics of disease causation
pathophysiology and psychodynamics of disease causationpathophysiology and psychodynamics of disease causation
pathophysiology and psychodynamics of disease causation
Preet Kaur
 
Lifestyles affecting cell disorders
Lifestyles affecting cell disordersLifestyles affecting cell disorders
Lifestyles affecting cell disordersbernadettevania
 
Basic Introduction of the General Pathology
Basic Introduction of the General PathologyBasic Introduction of the General Pathology
Basic Introduction of the General Pathology
Jai Kumar Panthi
 
Session 5: How Environmental Toxins are Linked to Metabolic Disorders and Chr...
Session 5: How Environmental Toxins are Linked to Metabolic Disorders and Chr...Session 5: How Environmental Toxins are Linked to Metabolic Disorders and Chr...
Session 5: How Environmental Toxins are Linked to Metabolic Disorders and Chr...
Center for Environmental Health
 
Basic definition and types of toxicology
Basic definition and types of toxicologyBasic definition and types of toxicology
Basic definition and types of toxicology
AbhishekJoshi312
 
Concepts of toxicology
Concepts of toxicologyConcepts of toxicology
Concepts of toxicology
Govt.college,Nagda, ujjain.M.P
 
Pathophysiology and pshycodyanamics (1)
Pathophysiology and pshycodyanamics (1)Pathophysiology and pshycodyanamics (1)
Pathophysiology and pshycodyanamics (1)
Gurdeep Arora
 
Basic Principle to Cell Injury and Cell Adaptation.pptx
Basic Principle to Cell Injury and Cell Adaptation.pptxBasic Principle to Cell Injury and Cell Adaptation.pptx
Basic Principle to Cell Injury and Cell Adaptation.pptx
70RahulMolla
 
introductionofpathology-1903613134414.pdf
introductionofpathology-1903613134414.pdfintroductionofpathology-1903613134414.pdf
introductionofpathology-1903613134414.pdf
yahomed519
 
Introduction of pathology
Introduction of pathologyIntroduction of pathology
Introduction of pathology
MAHMOUD IBRAHIM
 
CELL INJURY, CELLULAR ADAPTATIONS, MECHANISM OF CELL I NJURY
CELL INJURY, CELLULAR ADAPTATIONS, MECHANISM OF CELL I NJURYCELL INJURY, CELLULAR ADAPTATIONS, MECHANISM OF CELL I NJURY
CELL INJURY, CELLULAR ADAPTATIONS, MECHANISM OF CELL I NJURY
Mithun Venugopal
 
CELL INJURY, CELLULAR ADAPTATIONS, MECHANISM OF CELL I NJURY
CELL INJURY, CELLULAR ADAPTATIONS, MECHANISM OF CELL I NJURYCELL INJURY, CELLULAR ADAPTATIONS, MECHANISM OF CELL I NJURY
CELL INJURY, CELLULAR ADAPTATIONS, MECHANISM OF CELL I NJURY
Mithun Venugopal
 
Environmental and Chemical Toxicology
Environmental and Chemical ToxicologyEnvironmental and Chemical Toxicology
Environmental and Chemical Toxicology
halavathramesh2
 
Theories of Aging
Theories of AgingTheories of Aging
Theories of Aging
Pradeep Singh Narwat
 
Animals in ResearchThe Importance of Animals in the Sc.docx
Animals in ResearchThe Importance of Animals in the Sc.docxAnimals in ResearchThe Importance of Animals in the Sc.docx
Animals in ResearchThe Importance of Animals in the Sc.docx
justine1simpson78276
 

Similar to Toxicology (20)

Chapt08 Lecture
Chapt08 LectureChapt08 Lecture
Chapt08 Lecture
 
BIOLSCI-1-1
BIOLSCI-1-1BIOLSCI-1-1
BIOLSCI-1-1
 
BIOLSCI-1-1
BIOLSCI-1-1BIOLSCI-1-1
BIOLSCI-1-1
 
Glutathione
GlutathioneGlutathione
Glutathione
 
A current genetic and epigenetic view on human
A current genetic and epigenetic view on humanA current genetic and epigenetic view on human
A current genetic and epigenetic view on human
 
pathophysiology and psychodynamics of disease causation
pathophysiology and psychodynamics of disease causationpathophysiology and psychodynamics of disease causation
pathophysiology and psychodynamics of disease causation
 
Lifestyles affecting cell disorders
Lifestyles affecting cell disordersLifestyles affecting cell disorders
Lifestyles affecting cell disorders
 
Basic Introduction of the General Pathology
Basic Introduction of the General PathologyBasic Introduction of the General Pathology
Basic Introduction of the General Pathology
 
Session 5: How Environmental Toxins are Linked to Metabolic Disorders and Chr...
Session 5: How Environmental Toxins are Linked to Metabolic Disorders and Chr...Session 5: How Environmental Toxins are Linked to Metabolic Disorders and Chr...
Session 5: How Environmental Toxins are Linked to Metabolic Disorders and Chr...
 
Basic definition and types of toxicology
Basic definition and types of toxicologyBasic definition and types of toxicology
Basic definition and types of toxicology
 
Concepts of toxicology
Concepts of toxicologyConcepts of toxicology
Concepts of toxicology
 
Pathophysiology and pshycodyanamics (1)
Pathophysiology and pshycodyanamics (1)Pathophysiology and pshycodyanamics (1)
Pathophysiology and pshycodyanamics (1)
 
Basic Principle to Cell Injury and Cell Adaptation.pptx
Basic Principle to Cell Injury and Cell Adaptation.pptxBasic Principle to Cell Injury and Cell Adaptation.pptx
Basic Principle to Cell Injury and Cell Adaptation.pptx
 
introductionofpathology-1903613134414.pdf
introductionofpathology-1903613134414.pdfintroductionofpathology-1903613134414.pdf
introductionofpathology-1903613134414.pdf
 
Introduction of pathology
Introduction of pathologyIntroduction of pathology
Introduction of pathology
 
CELL INJURY, CELLULAR ADAPTATIONS, MECHANISM OF CELL I NJURY
CELL INJURY, CELLULAR ADAPTATIONS, MECHANISM OF CELL I NJURYCELL INJURY, CELLULAR ADAPTATIONS, MECHANISM OF CELL I NJURY
CELL INJURY, CELLULAR ADAPTATIONS, MECHANISM OF CELL I NJURY
 
CELL INJURY, CELLULAR ADAPTATIONS, MECHANISM OF CELL I NJURY
CELL INJURY, CELLULAR ADAPTATIONS, MECHANISM OF CELL I NJURYCELL INJURY, CELLULAR ADAPTATIONS, MECHANISM OF CELL I NJURY
CELL INJURY, CELLULAR ADAPTATIONS, MECHANISM OF CELL I NJURY
 
Environmental and Chemical Toxicology
Environmental and Chemical ToxicologyEnvironmental and Chemical Toxicology
Environmental and Chemical Toxicology
 
Theories of Aging
Theories of AgingTheories of Aging
Theories of Aging
 
Animals in ResearchThe Importance of Animals in the Sc.docx
Animals in ResearchThe Importance of Animals in the Sc.docxAnimals in ResearchThe Importance of Animals in the Sc.docx
Animals in ResearchThe Importance of Animals in the Sc.docx
 

Recently uploaded

Ophthalmology Clinical Tests for OSCE exam
Ophthalmology Clinical Tests for OSCE examOphthalmology Clinical Tests for OSCE exam
Ophthalmology Clinical Tests for OSCE exam
KafrELShiekh University
 
The POPPY STUDY (Preconception to post-partum cardiovascular function in prim...
The POPPY STUDY (Preconception to post-partum cardiovascular function in prim...The POPPY STUDY (Preconception to post-partum cardiovascular function in prim...
The POPPY STUDY (Preconception to post-partum cardiovascular function in prim...
Catherine Liao
 
For Better Surat #ℂall #Girl Service ❤85270-49040❤ Surat #ℂall #Girls
For Better Surat #ℂall #Girl Service ❤85270-49040❤ Surat #ℂall #GirlsFor Better Surat #ℂall #Girl Service ❤85270-49040❤ Surat #ℂall #Girls
For Better Surat #ℂall #Girl Service ❤85270-49040❤ Surat #ℂall #Girls
Savita Shen $i11
 
Triangles of Neck and Clinical Correlation by Dr. RIG.pptx
Triangles of Neck and Clinical Correlation by Dr. RIG.pptxTriangles of Neck and Clinical Correlation by Dr. RIG.pptx
Triangles of Neck and Clinical Correlation by Dr. RIG.pptx
Dr. Rabia Inam Gandapore
 
The Normal Electrocardiogram - Part I of II
The Normal Electrocardiogram - Part I of IIThe Normal Electrocardiogram - Part I of II
The Normal Electrocardiogram - Part I of II
MedicoseAcademics
 
24 Upakrama.pptx class ppt useful in all
24 Upakrama.pptx class ppt useful in all24 Upakrama.pptx class ppt useful in all
24 Upakrama.pptx class ppt useful in all
DrSathishMS1
 
Phone Us ❤85270-49040❤ #ℂall #gIRLS In Surat By Surat @ℂall @Girls Hotel With...
Phone Us ❤85270-49040❤ #ℂall #gIRLS In Surat By Surat @ℂall @Girls Hotel With...Phone Us ❤85270-49040❤ #ℂall #gIRLS In Surat By Surat @ℂall @Girls Hotel With...
Phone Us ❤85270-49040❤ #ℂall #gIRLS In Surat By Surat @ℂall @Girls Hotel With...
Savita Shen $i11
 
Physiology of Special Chemical Sensation of Taste
Physiology of Special Chemical Sensation of TastePhysiology of Special Chemical Sensation of Taste
Physiology of Special Chemical Sensation of Taste
MedicoseAcademics
 
Charaka Samhita Sutra Sthana 9 Chapter khuddakachatuspadadhyaya
Charaka Samhita Sutra Sthana 9 Chapter khuddakachatuspadadhyayaCharaka Samhita Sutra Sthana 9 Chapter khuddakachatuspadadhyaya
Charaka Samhita Sutra Sthana 9 Chapter khuddakachatuspadadhyaya
Dr KHALID B.M
 
New Directions in Targeted Therapeutic Approaches for Older Adults With Mantl...
New Directions in Targeted Therapeutic Approaches for Older Adults With Mantl...New Directions in Targeted Therapeutic Approaches for Older Adults With Mantl...
New Directions in Targeted Therapeutic Approaches for Older Adults With Mantl...
i3 Health
 
KDIGO 2024 guidelines for diabetologists
KDIGO 2024 guidelines for diabetologistsKDIGO 2024 guidelines for diabetologists
KDIGO 2024 guidelines for diabetologists
د.محمود نجيب
 
Non-respiratory Functions of the Lungs.pdf
Non-respiratory Functions of the Lungs.pdfNon-respiratory Functions of the Lungs.pdf
Non-respiratory Functions of the Lungs.pdf
MedicoseAcademics
 
Antiulcer drugs Advance Pharmacology .pptx
Antiulcer drugs Advance Pharmacology .pptxAntiulcer drugs Advance Pharmacology .pptx
Antiulcer drugs Advance Pharmacology .pptx
Rohit chaurpagar
 
THOA 2.ppt Human Organ Transplantation Act
THOA 2.ppt Human Organ Transplantation ActTHOA 2.ppt Human Organ Transplantation Act
THOA 2.ppt Human Organ Transplantation Act
DrSathishMS1
 
ANATOMY AND PHYSIOLOGY OF URINARY SYSTEM.pptx
ANATOMY AND PHYSIOLOGY OF URINARY SYSTEM.pptxANATOMY AND PHYSIOLOGY OF URINARY SYSTEM.pptx
ANATOMY AND PHYSIOLOGY OF URINARY SYSTEM.pptx
Swetaba Besh
 
Lung Cancer: Artificial Intelligence, Synergetics, Complex System Analysis, S...
Lung Cancer: Artificial Intelligence, Synergetics, Complex System Analysis, S...Lung Cancer: Artificial Intelligence, Synergetics, Complex System Analysis, S...
Lung Cancer: Artificial Intelligence, Synergetics, Complex System Analysis, S...
Oleg Kshivets
 
Charaka Samhita Sutra sthana Chapter 15 Upakalpaniyaadhyaya
Charaka Samhita Sutra sthana Chapter 15 UpakalpaniyaadhyayaCharaka Samhita Sutra sthana Chapter 15 Upakalpaniyaadhyaya
Charaka Samhita Sutra sthana Chapter 15 Upakalpaniyaadhyaya
Dr KHALID B.M
 
BRACHYTHERAPY OVERVIEW AND APPLICATORS
BRACHYTHERAPY OVERVIEW  AND  APPLICATORSBRACHYTHERAPY OVERVIEW  AND  APPLICATORS
BRACHYTHERAPY OVERVIEW AND APPLICATORS
Krishan Murari
 
Surgical Site Infections, pathophysiology, and prevention.pptx
Surgical Site Infections, pathophysiology, and prevention.pptxSurgical Site Infections, pathophysiology, and prevention.pptx
Surgical Site Infections, pathophysiology, and prevention.pptx
jval Landero
 
ARTHROLOGY PPT NCISM SYLLABUS AYURVEDA STUDENTS
ARTHROLOGY PPT NCISM SYLLABUS AYURVEDA STUDENTSARTHROLOGY PPT NCISM SYLLABUS AYURVEDA STUDENTS
ARTHROLOGY PPT NCISM SYLLABUS AYURVEDA STUDENTS
Dr. Vinay Pareek
 

Recently uploaded (20)

Ophthalmology Clinical Tests for OSCE exam
Ophthalmology Clinical Tests for OSCE examOphthalmology Clinical Tests for OSCE exam
Ophthalmology Clinical Tests for OSCE exam
 
The POPPY STUDY (Preconception to post-partum cardiovascular function in prim...
The POPPY STUDY (Preconception to post-partum cardiovascular function in prim...The POPPY STUDY (Preconception to post-partum cardiovascular function in prim...
The POPPY STUDY (Preconception to post-partum cardiovascular function in prim...
 
For Better Surat #ℂall #Girl Service ❤85270-49040❤ Surat #ℂall #Girls
For Better Surat #ℂall #Girl Service ❤85270-49040❤ Surat #ℂall #GirlsFor Better Surat #ℂall #Girl Service ❤85270-49040❤ Surat #ℂall #Girls
For Better Surat #ℂall #Girl Service ❤85270-49040❤ Surat #ℂall #Girls
 
Triangles of Neck and Clinical Correlation by Dr. RIG.pptx
Triangles of Neck and Clinical Correlation by Dr. RIG.pptxTriangles of Neck and Clinical Correlation by Dr. RIG.pptx
Triangles of Neck and Clinical Correlation by Dr. RIG.pptx
 
The Normal Electrocardiogram - Part I of II
The Normal Electrocardiogram - Part I of IIThe Normal Electrocardiogram - Part I of II
The Normal Electrocardiogram - Part I of II
 
24 Upakrama.pptx class ppt useful in all
24 Upakrama.pptx class ppt useful in all24 Upakrama.pptx class ppt useful in all
24 Upakrama.pptx class ppt useful in all
 
Phone Us ❤85270-49040❤ #ℂall #gIRLS In Surat By Surat @ℂall @Girls Hotel With...
Phone Us ❤85270-49040❤ #ℂall #gIRLS In Surat By Surat @ℂall @Girls Hotel With...Phone Us ❤85270-49040❤ #ℂall #gIRLS In Surat By Surat @ℂall @Girls Hotel With...
Phone Us ❤85270-49040❤ #ℂall #gIRLS In Surat By Surat @ℂall @Girls Hotel With...
 
Physiology of Special Chemical Sensation of Taste
Physiology of Special Chemical Sensation of TastePhysiology of Special Chemical Sensation of Taste
Physiology of Special Chemical Sensation of Taste
 
Charaka Samhita Sutra Sthana 9 Chapter khuddakachatuspadadhyaya
Charaka Samhita Sutra Sthana 9 Chapter khuddakachatuspadadhyayaCharaka Samhita Sutra Sthana 9 Chapter khuddakachatuspadadhyaya
Charaka Samhita Sutra Sthana 9 Chapter khuddakachatuspadadhyaya
 
New Directions in Targeted Therapeutic Approaches for Older Adults With Mantl...
New Directions in Targeted Therapeutic Approaches for Older Adults With Mantl...New Directions in Targeted Therapeutic Approaches for Older Adults With Mantl...
New Directions in Targeted Therapeutic Approaches for Older Adults With Mantl...
 
KDIGO 2024 guidelines for diabetologists
KDIGO 2024 guidelines for diabetologistsKDIGO 2024 guidelines for diabetologists
KDIGO 2024 guidelines for diabetologists
 
Non-respiratory Functions of the Lungs.pdf
Non-respiratory Functions of the Lungs.pdfNon-respiratory Functions of the Lungs.pdf
Non-respiratory Functions of the Lungs.pdf
 
Antiulcer drugs Advance Pharmacology .pptx
Antiulcer drugs Advance Pharmacology .pptxAntiulcer drugs Advance Pharmacology .pptx
Antiulcer drugs Advance Pharmacology .pptx
 
THOA 2.ppt Human Organ Transplantation Act
THOA 2.ppt Human Organ Transplantation ActTHOA 2.ppt Human Organ Transplantation Act
THOA 2.ppt Human Organ Transplantation Act
 
ANATOMY AND PHYSIOLOGY OF URINARY SYSTEM.pptx
ANATOMY AND PHYSIOLOGY OF URINARY SYSTEM.pptxANATOMY AND PHYSIOLOGY OF URINARY SYSTEM.pptx
ANATOMY AND PHYSIOLOGY OF URINARY SYSTEM.pptx
 
Lung Cancer: Artificial Intelligence, Synergetics, Complex System Analysis, S...
Lung Cancer: Artificial Intelligence, Synergetics, Complex System Analysis, S...Lung Cancer: Artificial Intelligence, Synergetics, Complex System Analysis, S...
Lung Cancer: Artificial Intelligence, Synergetics, Complex System Analysis, S...
 
Charaka Samhita Sutra sthana Chapter 15 Upakalpaniyaadhyaya
Charaka Samhita Sutra sthana Chapter 15 UpakalpaniyaadhyayaCharaka Samhita Sutra sthana Chapter 15 Upakalpaniyaadhyaya
Charaka Samhita Sutra sthana Chapter 15 Upakalpaniyaadhyaya
 
BRACHYTHERAPY OVERVIEW AND APPLICATORS
BRACHYTHERAPY OVERVIEW  AND  APPLICATORSBRACHYTHERAPY OVERVIEW  AND  APPLICATORS
BRACHYTHERAPY OVERVIEW AND APPLICATORS
 
Surgical Site Infections, pathophysiology, and prevention.pptx
Surgical Site Infections, pathophysiology, and prevention.pptxSurgical Site Infections, pathophysiology, and prevention.pptx
Surgical Site Infections, pathophysiology, and prevention.pptx
 
ARTHROLOGY PPT NCISM SYLLABUS AYURVEDA STUDENTS
ARTHROLOGY PPT NCISM SYLLABUS AYURVEDA STUDENTSARTHROLOGY PPT NCISM SYLLABUS AYURVEDA STUDENTS
ARTHROLOGY PPT NCISM SYLLABUS AYURVEDA STUDENTS
 

Toxicology

  • 1. See discussions, stats, and author profiles for this publication at: https://www.researchgate.net/publication/266738818 A Handbook of MCQs in Toxicology Book · May 2014 CITATIONS 0 READS 50,587 All content following this page was uploaded by Wajhul Qamar on 25 November 2019. The user has requested enhancement of the downloaded file.
  • 2. Preface The goal of this book is to provide comprehensive multiple choice questions from the field of toxicological science. It also includes an intention to provide a brain storming session for the individuals associated with toxicology. The present book “A Handbook of MCQs in Toxicology” is targeted towards students and general audiences. This book provides an ample number of toxicological questions for the students preparing for their examinations or simply for testing and practicing knowledge of toxicology. The book will also help those who want to improve their toxicological knowledge in a smaller amount of time. With the best wishes this book is presented to those interested in toxicology. Wajhul Qamar, Ph.D. Assistant Professor Research Center, College of Pharmacy King Saud University Riyadh, Saudi Arabia
  • 4. "All substances are poisons: there is none which is not a poison. The right dose differentiates a poison and a remedy." Paracelsus (1493-1541)
  • 5. Introduction Every living entity has an interaction with other livings or non-livings for their appropriate survival. These kinds of interactions are essential as organisms are dependent on other living and non-living matter to obtain energy and achieve growth so that they could fulfill all the requirements to be alive. Non- living matter play an irreplaceable role in normal physiology of the living beings. Various enzymes play their essential role only in the presence of a cofactor, which are mainly inorganic metal ions including Mg++, Mn++, Zn++ etc. Ca++ also plays an essential role in cellular signaling and bone formation; these are to mention a few. Other components that are involved in physiology include selenium, sodium, potassium, iron etc. and organisms require all these elements on a daily basis through food and water. Oxygen is also an essential element that play major role in oxidative metabolism and generation of energy for the daily need of the organism. But all of above mentioned or other components not mentioned, are needed in a particular dose1. Uptake of these components above or below the required dose can potentially alter the physiology and exhibit a condition that is a disease or ailment. In the same manner a substance that is not required to the organism can have adverse physiological alterations and severe effects after an exposure. These unwanted substances mainly include a variety of chemicals; others are biological (e.g. viruses, bacteria etc.) or physical (e.g. UV radiations) agents. Agents that impose such kind of adverse effects on the organisms are known as xenobiotics or toxicants and effects are called toxicities. 1 Amount of a substance in mg/kg body weight, which exerts its effect on the organism.
  • 6. Dose, of a particular substance, is an important factor in determining whether a substance will have an adverse effect on the organism or be eliminated from the body without any effect. Dose also depends on the route of exposure, through which a substance gets entry into the biological system. Most of the xenobiotics cause an obvious alteration in redox status of the cell or tissue by accelerating free radical generation processes or active metabolite formation of the xenobiotics itself. Moreover, it may alter the expression of various genes and proteins involved in normal cell growth and functioning. In this context a toxicological agent is capable of interfering with the normal physiological activities in an organism by any one or a combination of the above mentioned interactions. In fact the generation of free radicals that is involved in oxidative cellular injuries and alterations of gene function (either genetically or epigenetically) are not two separable events. They occur simultaneously. In other words, the effect a xenobiotics exhibit in a biological system is due to a combination of genetic and extragenetic interactions within the cell. Inflammation is the most common response in higher animals when there is an exposure to a toxic agent. It is simply a protective response but persistence for a longer period (a chronic condition) may result into a variety of diseases. Chronic Obstructive Pulmonary Disease (COPD), pulmonary fibrosis, Arthritis etc. are the examples of chronic2 inflammatory diseases which may result after repeated episodes of acute inflammation caused by a xenobiotics. Moreover, chronic inflammation is also involved in cancer development. 2 A condition, which persists for a period of more than three months. In most of the chronic diseases the period may be in years. Various xenobiotics mimic the function of and interfere with the biosynthesis of hormones which are essential for the normal physiological functions, growth and reproduction. This kind of xenobiotics is known as endocrine disruptors. Exposure to such kind of chemicals is a matter of concern as they show their effects at very low doses. Release of these chemicals into environment is, sometime, unavoidable as some of them are used as pesticides, in plastic industry and other chemical manufacturing units. Xenobiotics under the category of mutagens and carcinogens3 are capable of altering the DNA sequences in such a way that the cells lose their regulation of cell division and proliferate in an uncontrolled manner. This condition can lead to tumor formation and metastasis. These effects are lethal and a major cause of deaths worldwide. Alterations in neurophysiology can be responsible for debilities in cognitive function and behavioral alterations. Neurotoxic4 agents are capable of inducing such kind of deleterious effects and can lead to development of neurodegenerative diseases like Alzheimer’s and Parkinson’s. They can affect peripheral or central nervous system and can induce different neuropathies depending on the part affected. These effects are mainly associated with low level of exposure. High level of exposure to neurotoxicants can lead to coma and sudden death. 3 All carcinogens are mutagens, but all mutagens are not carcinogens. This is because every mutation does not result in cancer. 4 Agents which cause damages to nervous tissue and disrupts central and peripheral nervous systems.
  • 7. There is an ample generation of endogenous toxic metabolites5, like ammonia, CO2 etc. to cause serious toxicities if not excreted out immediately. Excretory processes are an integral part of the body functioning. Kidneys are the main organ involved in excretion of not only endogenous harmful metabolites but also of the xenobiotics and their metabolites. Lungs are mainly involved in exhalation of CO2 and other volatile substances such as ethanol. Liver is involved in xenobiotics metabolism and also in excretion of certain kind of xenobiotics through bile. Injury to these organs by a xenobiotic would hamper the excretion of endogenous metabolites at one end, and debility in their other essential functions at the other. The overall scenario depicts a condition, exposure to xenobiotics, which is not totally avoidable. The nature always tries to make a balance between degeneration and protection6, but it is our own activities that are responsible for the tilting of this balance towards degeneration. 5 Normal metabolic processes release unwanted byproducts which can be toxic to the body if not excreted out. 6 Various defensive tools exist in the body in form of detoxifying enzymes (like Cytochrome P450, Alcohol dehydrogenase etc.), antioxidant enzymes (Glutathione reductase, catalase, superoxide dismutase etc.) and reducing equivalents (glutathione, NADPH, thioredoxin, glutaredoxin, FADH etc.). Multiple Choice Questions (MCQs) Multiple choice questions provide a great deal of revision of a particular subject area, whenever there is a need. The present book contains a number of MCQs from toxicology and divided into sets of 20 MCQs each. At the end of every set the answer keys are given. In addition to the answer keys, whenever it was needed a brief explanation of the answer is also given. Full explanations of all the positive or negative answers are out of scope of this book.
  • 9. MCQs Set 1 1. Which of the following case(s) can be categorized as toxicity? A Development of chronic lung inflammation in smokers. B Chronic rhinitis associated with influenza. C Gout, which is inflammation of joints due to accumulation of uric acid crystals. D Post-surgical pus formation in a wound on forearm. E All of the above 2. Liver is the main metabolizing organ in the body. Which of the following statement is true in association with toxicant metabolism and reduction of toxicity? A Liver alone metabolizes all the toxicants and does not need any further assistance B Kidney plays an essential role in elimination of metabolized toxicants through urine. C Cardiovascular system plays an important role in detoxification of the chemicals. D All the chemicals are removed from the body through bile, after metabolism in the liver. E Gastrointestinal tract controls the flow of chemicals into the liver. 3. Which of the following is not a natural rout of exposure to chemicals? A Inhalation B Oral/Gastrointestinal tract C Topical/Dermal D Intraperitoneal E None of the above
  • 10. 4. Which one of the following is most accurate in case of biotransformation of the chemicals/toxicants? A It is a synonym of the metabolism. B Biotransformation always results in the activation of a neutral chemical. C Biotransformation is the transformation of one chemical into the other irrespective of its activity. D Biotransformation reduces toxicity by transforming chemicals into neutral ones. E Biotransformation does not change the chemicals, but alters their activities in a biological system. 5. What is bioaccumulation? A Accumulation of biological entities in a geographical area. B Accumulation of fossils of plants and animals underneath several layers of mud, rock and the sand. C Accumulation of fluid in a body part after a toxic exposure. D Accumulation of chemicals in body mainly in fat tissue and bones. E Cumulative effect of several toxicants on biological system. 6. Bioactivation encompasses the following. A A less harmful substance is converted into more harmful one. B It is a process which activates the metabolizing enzymes in the liver. C Bioactivation is a part of biotransformation process. D B & C E A & C 7. Which of the following enzymes play crucial role in detoxification of the chemicals? A Digestive enzymes in the gastrointestinal tract destroy all the chemicals entering into the body. B Cytochrome p450 oxidases, UDP- glucuronyltransferases and glutathione S-transferases C Superoxide dismutase, catalase, glutathione peroxidase D B & C E All of the above 8. Chemicals have specific targets in the body. A True B Few does have targets, others are nonspecific C False D Depends on the route of exposure E Chemicals decide after entering into the body. 9. Which is an acute event of toxicity? A Lung inflammation after two days of inhalation of metal dust. B Liver toxicity after three daily doses of 325 mg of paracetamol for one week only. C Liver toxicity of three doses of 500 mg of paracetamol for one day only. D A toxic event occurring suddenly without an unknown cause. E A & C
  • 11. 10. Which is true regarding chronic exposure? A Repeated exposure occurring for a period of three months. B Repeated exposure occurring for a period of one month. C Repeated exposure occurring for a period of more than three months. D Repeated exposure occurring for a period of one day. E It is not categorized as chronic if no toxicity occurs. 11. Following phases are included in biotransformation of the chemicals. A Phase 0, Phase I, Phase II B Phase I, Phase II and sometime Phase III C Phase 0, Phase I, Phase II, Phase III and Phase IV D Phase 0, Phase I, E Phase 0, Phase I, Phase II, Phase III 12. Chemicals are toxic at any dose level. A A few chemicals, which are lethal to humans, can be defined like that. B No, dose determines whether they are toxic. C There is no dose response relationship when we talk about highly toxic chemicals. D The statement can be applied to inhalant toxicants, because of lungs’ high sensitivity. E This is true for all kind of pesticides. 13. Which of the following will have the most damaging effects? A A hydrophilic chemical via inhalation route. B A lipophilic chemical via dermal route. C A lipophilic chemical via inhalation route. D A hydrophilic chemical via oral route. E A hydrophilic chemical via dermal route. 14. Which of the following will have the least damaging effects? A A hydrophilic chemical via inhalation route. B A lipophilic chemical via dermal route. C A lipophilic chemical via inhalation route. D A hydrophilic chemical via oral route. E A hydrophilic chemical via dermal route. 15. Most of the chemicals are excreted out by the kidneys through urine. Which are the other organs or modes involved in excretion of chemicals? A Lungs through expiration of volatile compounds. B Sweat glands excrete a number of metabolites out of the body. C Through bile. D Mammary glands E All of the above but A, B and C are important 16. What is the biological half-life (t1/2) of a chemical? A A chemical reduces the life of an individual to half of the average life. B The time in which systemic concentration of a chemical reduces to half of its initial concentration. C Half of the time taken to reach the maximum concentration of chemical in the system. D Half of the dose of a chemical which can affect the quality of life. E None of the above 17. Which information can be obtained from an acute toxicity study? A Median toxic dose (TD50) B Median lethal dose (LD50) C No Observed Adverse Effect Level (NOEL) D Target organ E All of the above
  • 12. 18. A particular dose of a substance X is minimal toxic to animal. Substance Y is also minimal toxic to the animals at the same dose, but when both the substances are administered together they show the toxicity several orders of magnitude higher than compared with individual administrations. This is an example of: A Potentiation B Synergism C Additivity D Acute Toxicity E Agonism 19. 1 ppm is equivalent to: A 1 g/kg B 1 μg/kg C 1 mg/kg D 1 mg/100g E None of the above 20. Which is true about the LD50? A Dose of a substance to which 50% of animals do not show any response. B Dose of a substance which kills 50% of animals exposed. C 50% of the dose of a substance which can kill an animal. D Dose of a substance which can kill 50 animals. E 50 mg/kg dose of a substance to test the toxic responses in animals. Answer keys MCQs Set 1: 1. A 2. B 3. D – Intraperitoneal route of administration is used in various experimental and clinical administrations where a test substance/drug is delivered into the peritoneal cavity with the help of a syringe and needle. 4. C 5. D 6. E 7. B 8. B 9. E 10. C 11. B 12. B 13. C 14. E 15. E – Theoretically toxicants/metabolites can be eliminated along with any secretion from the body. 16. B 17. E – In an acute toxicity study any end point parameter such as death, brain damage, liver damage etc. can be used to obtain information regarding a particular toxicant. 18. B – In potentiation a non-toxic substance increases the toxicity of a relatively less toxic substance. In synergism a substance shows unusually high toxicity in presence of a less toxic substance, it is much greater than additive effect. In agonism a substance (agonist) mimics the mechanism of action of other substance, its opposite is known as antagonism. 19. C – 1 mg is one millionth part of a kilogram. 20. B
  • 14. MCQs Set 2 1 Acceptable daily intake (ADI) A An estimate of amount of food taken daily. B Amount of nutrients required on daily basis. C A & B D An estimate of the amount of substance in the food that can be ingested daily over a lifetime by humans without appreciable health risk. E All of the above 2 Descriptive toxicology is concerned directly with A Description of toxicology related research fields. B Origin of toxicants in nature. C Toxicity testing, which provides information for safety evaluation and regulatory requirements? D Describing the toxicology as a science and art to laymen and unrelated scientific fields. E A and D 3 The term ‘toxin’ generally refers to toxic substances that are _____ A Any kind of poisons. B Produced by biological systems such as plants, animals, fungi, or bacteria. C Toxicants released as industrial effluents. D Toxic elements of inorganic origin such as mercury, lead, arsenic etc. E All substances except gases.
  • 15. 4 The term ‘toxicant’ is used in speaking of toxic substances that are A Specifically toxic to humans. B Nonlethal in their effects. C produced by organisms. D produced by anthropogenic activities. E Toxic to organisms such as plants, animals, fungi or bacteria. 5 What is chemical allergy? A The allergy described in terms of chemistry. B An immunologically mediated adverse reaction to a chemical resulting from previous sensitization to that chemical or to a structurally similar one. C When an individual exhibit adverse immunological response against all chemicals. D Immunological response where chemicals mediate the process instead of antibodies. E A and D 6 Chemical idiosyncrasy A Extreme sensitivity to low doses or extreme insensitivity to high doses of the chemical. B Attribute of chemicals where they can elicit low or high response that solely depends on their chemistry. C Abnormal responses of individuals towards chemicals which is determined by genetic constitution. D Abnormal responses of individuals towards chemicals which is determined by environmental factors. E A and C 7 Threshold dose A The dose level at which mortality starts occurring. B A threshold for lethal dose. C The dose below which no effects appear. D A and B E The dose at which 50% animals exhibit toxicity signs 8 No observed adverse effect level (NOAEL) A The condition of experimental animals when no adverse effect can be observed. B The maximum dose at which the toxicant show no signs of toxicity. C A condition where adverse effects exist, but are hard to observe. D A and C E The maximum dose of a chemical which caused the death, hence no other adverse effect got observed. 9 Which of the following can be a source of toxicants? A Car B Fruits C Cosmetics D Medicines E All of the above 10 Which one of the following elements is required by our body for normal functioning of some enzymes? A Mercury (Hg) B Zinc (Zn) C Lead (Pb) D Antimony (Sb) E Scandium (Sc)
  • 16. 11 Which one of the elements is not required by our body? A Selenium (Se) B Potassium (K) C Copper (Cu) D Arsenic (As) E Phosphorus (P) 12 What is a reference dose (RfD)? A Dose of a standard chemical to test the toxic doses of other chemicals. B An estimate of exposure to an agent for a long period without any appreciable risk to life. C Dose of an agent during a previous exposure to estimate the risks of a latest exposure to the same agent. D All of the above E None of the above 13 Where in the body do toxicants/chemicals get stored? A Plasma proteins B Body fat C Liver and kidneys D Bones E All of the above 14 The mode of excretion of xenobiotics from the body. Select which applies. A Urinary B Fecal (nonabsorbed, biliary excretion) C Exhalation D Sweat, saliva and milk E All of the above 15 Egg shell thinning is caused by the pesticides DDT and DDE. The mode of exposure in this case is the following: A Direct exposure of birds to DDT and DDE. B Nesting of birds in DDT and DDE exposed agricultural fields. C Movement of DDT and DDE in food chain. D DDT and DDE don’t cause egg shell thinning. E Direct exposure of eggs to DDT and DDE. 16 Which one of the following does not elicit toxicological interests? A Forest fire B Volcanic eruption C Earthquake D Acid rain E Sand storm 17 What does IDLH stands for? A International Directory For Long Term Health Effects B Immediately Dangerous To Lung Or Heart C International Directory For Lung And Heart Diseases D Immediate Or Delayed Effects On Lung Or Heart E Immediately Dangerous To Life Or Health 18 Which one of the following are the main targets of lead toxicity? A Liver and kidneys B Nervous system and hematopoietic system C Heart and lung D Bones and muscles E Skin and hair
  • 17. 19 Organic mercury targets_________ whereas inorganic mercury primarily targets__________. A Bones, ligaments B Liver, hematopoietic system C Nervous system, kidneys D Hematopoietic system, nervous system E Liver, nervous system 20 ‘Itai-itai’ disease is caused by______ A Cadmium B Mercury C Lead D Copper E Chromium Answer keys MCQs Set 2: 1. D 2. C 3. B 4. D 5. B 6. E 7. C 8. B 9. E 10. B 11. D – Arsenic is one of the most toxic substances and people are exposed to it through contaminated water and soil. 12. B 13. E – Storage of toxicants within the body depends on their affinity towards the different kinds of tissues and their components. For example lipophilic substances are most likely to be stored in fat tissue. 14. E 15. C 16. C 17. E – represents the levels of a particular hazardous substance. 18. B – Lead targets developing nervous system and shows severe effects in children. In adults mainly hematopoietic system is targeted. 19. C 20. A – Itai itai is a cadmium poisoning disaster occurred around 1912 in Japan. The disease affected kidneys and bones causing severe pain in joints and spine, hence the name itai itai (“Ouch, Ouch” or “It hurts, it hurts”).
  • 19. MCQs Set 3 1 ‘Black foot’ disease is caused by ________ A Chromium B Mercury C Arsenic D Lead E Copper 2 Which of the following is not a pesticide? A Chloropicrin B Fluoroacetamide C Malachite green D Nicotine E Malathion 3 Which of the following toxins is not produced by bacteria? A Cholera toxin B Aflatoxin C Botulinum toxin D Tetanus toxin E Diphtheria toxin 4 Ergotism is associated with toxin produced by __________ A Bacteria B Plant C Fungus D Virus E Mycoplasma
  • 20. 5 Aspergillus species produce aflatoxins. Aflatoxin B1, one of the aflatoxins, causes following toxic effect/s in humans: A Severe liver toxicity and Carcinogenesis B It is less toxic due to its natural origin C Severe renal effects resulting in kidney failure D Cardiotoxicity E Respiratory collapse 6 Tetrodotoxin, a deadly toxin, is found in _________ A Rattle snake B Shellfish C Mussels D Puffer fish E Scorpion 7 Which is the most likely toxic effect of cosmetics? A Skin corrosion B Allergic contact dermatitis C Local muscular degeneration D Local neuronal damage E Major risk of systemic toxicity after absorption of chemicals through skin. 8 Gastrointestinal tract does not have any profound effect on nature of ingested chemicals A True B False C Maybe D Gut-microflora plays important role in biotransformation of ingested chemicals E It does not matter whether gastrointestinal tract have any effects on chemicals 9 Which of the following is the main way of transportation of a lipid soluble toxicant within body? A Filtration B Special transport C Endocytosis D A and C E Passive diffusion 10 What is the most common toxicity target of ethanol (beverage alcohol) in humans? A Brain B Fetus C Liver D Kidneys E Heart 11 What are the common targets of ethanol toxicity in humans? A Liver, brain, heart and kidneys B Liver, lungs and intestine C Liver, brain and fetus D Spleen, liver, brain and thymus E All of the above 12 Methanol consumption is associated with permanent blindness. Following comment is true regarding methanol toxicity. A Methanol, unlike ethanol, causes direct toxicity to optical nerves after absorption. B Apart from the blindness, methanol is less toxic than ethanol. C Alcohol dehydrogenase converts methanol into formaldehyde which causes blindness. D Aldehyde dehydrogenase converts methanol into formaldehyde which causes blindness. E Alcohol dehydrogenase and aldehyde dehydrogenase
  • 21. converts methanol into formic acid which causes blindness. 13 Which part of the body is primarily affected by caffeine intoxication? A Heart B Brain C Liver D Kidneys E Gastrointestinal tract 14 Leaves from the following plants are the significant source of caffeine. A Coffea arabica B Erythroxylum coca C Cola acuminata D Camellia sinensis E Theobroma cacao 15 Who first of all described the association between chemical exposure and cancer? A Percival Pott B Paracelsus C Mathieu Orfila D Bernardino Ramazzini E Oswald Schmiedeberg 16 Who is the father of forensic toxicology? A Louis Lewin B Paracelsus C Mathieu Orfila D Bernardino Ramazzini E Oswald Schmiedeberg 17 Who is the father of occupational medicine? A Louis Lewin B Paracelsus C Mathieu Orfila D Bernardino Ramazzini E Oswald Schmiedeberg 18 Exposure to __________ is associated with occupation. A Aflatoxins B Cigarette smoke C Formaldehyde D Ethanol E Acetaminophen 19 Exposure to __________ is associated with lifestyle. A Chromium B Benzene C Benzidine D Nicotine E Asbestos 20 Mesothelioma (cancer of lining covering internal organs) is associated with exposure to ____ A Benzene B Nickel C Mercury D Arsenic E Asbestos
  • 22. Answer keys MCQs Set 3: 1. C 2. C - Malachite green is used as a dye in leather and paper industries. 3. B 4. C 5. A 6. D – Liver, intestine and skin of puffer fish is known to be source of tetrodotoxin, a potent neurotoxin, which is produced by symbiotic bacteria Pseudoalteromonas tetraodonis and species of Vibrio and Pseudomonas. 7. B 8. D 9. E 10. C 11. C – After liver and brain, the ethanol causes fetal toxicity known as Alcohol Fetal Syndrome (AFS) causing severe mental and physical damages in developing babies. 12. E 13. B 14. D 15. A 16. C 17. D 18. C – Formaldehyde is used in shoe industry and has been associated with nasal cancer. 19. D 20. E TOXICOLOGY MCQs SET 4
  • 23. MCQs Set 4 1 Benzene is known to cause _________ cancer. A Bone marrow B Cervical C Ovarian D Prostate E Breast 2 Thalidomide is known for its ___________ effects. A Teratogenic B Carcinogenic C Mutagenic D Neurotoxic E Hepatotoxic 3 The very well reported birth defect of thalidomide is________ A Cleft palate B Congenital heart disease C Ectrodactyly D Club foot E Phocomelia 4 What is the expression of dose of a substance? A mg/lbs B g/kg C μg/100g D μg/kg E mg/kg
  • 24. 5 Ppm (parts per million) is equivalent to _________ A ml/L B mg/kg C μg/kg D mg/L E μg/g 6 Which of the following pesticides is acetylcholinesterase inhibitor? A Formamidines B DDT and DDE C Nicotine D Pyrethroids E Organophosphates 7 Which one is an antidote for organophosphate poisoning? A Activated charcoal B EDTA C Ipecac D Atropine E BAL 8 Dose is defined as the __________________________ A Amount of substance which is released into the environment. B Amount of substance which reaches the target site in the body. C Amount of substance which is converted into active metabolite to exert toxic effect. D Amount of substance which enters into the body. E Amount of substance is bound to the plasma proteins. 9 Which one of the following is true regarding Dose-response relationship? A Response is directly proportional to the dose of a substance. B Dose exhibits ‘all or none’ response. C The dose which elicit a toxic response. D The dose which does not induce any response. E None of the above. 10 What is a quantal dose-response? A Continuous responses in an individual against varying doses of a substance. B The responses which can be quantified. C The most probable responses in experimental animals after toxicant exposure. D The responses which cannot be quantified. E Individual responses distributed in a population to different doses of a substance. 11 Following is an example of quantal dose-response observation. A Toxic effects in response to increasing doses of a chemical. B Effect of a dietary supplement on body weight in a population. C Observation of median lethal dose (LD50) in rats. D Death of an individual after a snake bite. E B and C
  • 25. 12 What is a graded dose-response? A Continuous responses in an individual against varying doses of a substance. B Responses to a single exposure at different time interval. C Effects other than the major one against a dose of a substance. D Individual responses distributed in a population to different doses of a substance. E A and C 13 What is the antidote for methanol poisoning? A Atropine B British anti lewisite C Charcoal D Ethanol E Disodium EDTA 14 Which one of the following is not an air pollutant? A Ozone B Argon C Nitrogen oxides D Sulfur oxides E Hydrogen sulfide 15 What is the mode of action of carbon monoxide (CO)? A It reduces the oxygen absorption by directly or indirectly damaging the alveoli. B It destroys the red blood cells. C Combines with hemoglobin to reduce oxygen carrying capacity of RBCs. D It has a corrosive effect on airways, specially bronchi and bronchioles. E It alters the structure of the hemoglobin. 16 Sulfur dioxide (SO2) affects the respiratory system by _________ A Forming sulfurous acid in respiratory tract, resulting in irritation. B Reducing oxygen carrying capacity of blood. C Causing allergy D Carcinogenesis E Respiratory collapse 17 Which of the following participate in formation of photochemical smog? A Oxides of sulfur B Hydrogen C Ozone D Oxides of nitrogen E Carbon dioxide 18 What is the cause of acid rain? A Over production of acids in factories. B Increased environmental concentration of Oxides of nitrogen and sulfur due to air pollution. C Increased use of acid in household cleaning. D Increased aviation activities. E Carbon dioxide (CO2) and global warming. 19 Which of the following is not a possible route of exposure to a toxicant in the environment? A Inhalation B Ingestion C Dermal absorption D Transdermal E None of the above
  • 26. 20 What happens to DDT when it enters the body? A It is stored in the bones. B It is water soluble and easily excreted out from the body. C It is fat soluble and stored in fat tissue. D It is converted into an active metabolite. E It bypasses the metabolism and excreted as such. Answer keys MCQs Set 4: 1. A 2. A 3. E – Phocomelia, a condition where long limb bones are not formed in fetuses. Arms without long bones appear like flippers hence the condition is called ‘Flipper Arms’ a common condition caused by thalidomide. 4. E – Dose of a substance is always measured in mg/kg body weight. Where the measurement in case of toxic gases or fumes the exposure level is measured as concentration in ppm/m3. 5. B 6. E 7. D 8. D 9. A 10. E 11. E 12. A 13. D – Ethanol is agonist for methanol and competes with it for metabolism by alcohol dehydrogenase and aldehyde dehydrogenase, thus minimizes the formation of toxic metabolites of methanol, formaldehyde and formic acid. Alcohol Dehydrogenase Aldehyde Dehydrogenase Methanol ---------------> Formaldehyde ------------------> Formic acid (Causes blindness) Alcohol Dehydrogenase Aldehyde Dehydrogenase Ethanol ----------------> Acetaldehyde -------------------> Acetic acid 14. B 15. C – Carbon monoxide has around 200 times greater affinity for hemoglobin than oxygen.
  • 27. 16. A – SO2 reacts with aqueous layer on lung epithelium to form sulfurous acid. SO2 + H2O → H2SO3 17. D 18. B - Oxides of nitrogen and sulfur reacts with environmental moisture to form nitric acid and sulfuric acid respectively. 19. D – Transdermal route is used to deliver certain drugs in clinical set up. 20. C TOXICOLOGY MCQs SET 5
  • 28. MCQs Set 5 1 Who is most likely to exhibit toxic effects of environmental tobacco smoke (ETS). A A person with asthma B An obese teenager C An athlete D A child in 2nd grade E A driver who smokes 2 Which of the following pneumocyte clears the particles deposited in the lungs? A Type I pneumocyte B Type II pneumocyte C Clara cells D Fibroblasts E Alveolar macrophages 3 Emphysema is an inflammatory tissue damage, which can be caused by toxicants. Which organ is affected in case of emphysema? A Kidneys B Heart C Lung D Skin E Nervous system 4 Which protein is mainly damaged in emphysema? A Collagen B Elastin C Keratin D Albumin E globulin
  • 29. 5 Cigarette smoking is the main cause of Chronic Obstructive Pulmonary Disease, which is a combination of following ailments. A Emphysema, chronic bronchitis, asthma B Chronic bronchitis, bronchiolitis, edema C Asthma, tracheitis, necrosis D Emphysema, cough, sneezing E Allergic cough, rhinitis, lower respiratory infections 6 Which of the following statements is true? A Light cigarettes are safer than the regular ones. B Electronic cigarettes are less harmful to the lungs and heart. C Filtered cigarettes are way safer than the non-filtered cigarettes. D Herbal cigarettes are natural hence harmless. E None of the above 7 What does PM10 indicate? A Stage of a lung disease. B Size of inhalant particles enough to reach and be deposited in the alveoli. C Number of cigarettes per day to cause lung cancer in 10 years. D Measure of pollutants in a unit volume of inhalant air. E Degree of concentration of toxicants present in the air. 8 A plenty of free radicals and oxidants are released with cigarette smoke. What does these oxidative species actually do to the lungs? A They alter the secretion of surfactants which protect the lung epithelium. B They block clearance of particulate matter by inhibiting cilia movement. C They damage hemoglobin and reduce oxygen carrying capacity of the blood. D They can damage cellular macromolecules like DNA, proteins and lipids and exert severe cellular damages leading to several diseases. E A and B 9 Four of the following are the sources of hazard and one is a hazard. Which one is a hazard? A Asbestos mining B Chemical factory C Cigarette D Metal fumes E Automobiles 10 Which of the following is not helpful in determining the dose of a toxic substance? A Body weight B Animal species (including human) C Origin of toxicant D Chemico-biological nature of the toxicant E Body surface area 11 Following are the normal functions of the metals in the body except _________. A Calcium in bone formation B Phosphorus in ATP C Iron in hemoglobin D Lithium in sodium channels activities E Magnesium in enzyme functions
  • 30. 12 What are the effects of cellular hypoxia in case of carbon monoxide poisoning? A Alveolar collapse B Lung edema C Brain damage and cardiac dysrhythmia D Disrupted glycolysis E Disrupted hematopoiesis 13 Why activated charcoal is administered in case of oral poisoning? A It deactivates toxicants B It increases the metabolic processes of detoxification C It chemically detoxifies the toxicants D It chelates metal circulating in the blood E It reduces absorption of toxicants by adsorbing them 14 Which of the following is associated with acetaminophen toxicity? A Methanol B N-acetyl-p-benzoquinone imine C 3-(1-methylpyrrolidin-2-yl)pyridine D Benzo(a)pyrene E Diethyl nitrosamine 15 Who is the most susceptible to hepatotoxicity by high doses of acetaminophen (paracetamol)? A A child in second grade B An old lady C A person on fasting D An alcoholic E A factory worker 16 What kind of toxicity does ammonia cause? A Cardiotoxicity B Tissue corrosion C Neurotoxicity D Reduced gastric motility E hepatotoxicity 17 Which one of the arsenic compounds causes hemolysis? A Arsenic trioxide (As2O3) B Arsenic pentoxide (As2O5) C Arsenic pentafluoride (AsF5) D Arsenic trisulfide (As2S3) E Arsine (AsH3) 18 Asbestos is a group of naturally occurring silicates, which include ____________. A Chrysotile and anthophyllite B Amosite and actinolite C Crocidolite D Tremolite E All of the above 19 Sodium azide (NaN3) which is used in car air bags and as preservative in laboratories has a mechanism of toxicity similar to _________. A Aconite B Cyanide C Nicotine D Arsenic E Lithium
  • 31. 20 Botulinum toxin causes death by___________. A CNS depression and coma B Cardiac failure C Respiratory failure D Severe hemolysis E Renal failure Answer keys MCQs Set 5: 1. A – Asthma can be triggered by tobacco smoke and pose severe damage. Children are also prone to toxic insults caused by tobacco smoke. However, tobacco smoke is toxic for anyone exposed. 2. E 3. C – In emphysema lung alveoli are damaged, become enlarged and loose flexibility. It is one of the several conditions collectively known as chronic obstructive pulmonary disease (COPD). 4. B 5. A 6. E 7. B 8. D – Oxidants exert their effect by damaging cellular macromolecules. 9. D 10. C 11. D – Lithium is used as medicine in mental illnesses like depression, schizophrenia and bipolar disorder. 12. C 13. E 14. B - N-acetyl-p-benzoquinone imine is reactive metabolite of acetaminophen which is responsible for liver damage and other associated toxic effects. CYP 2E1 (Liver damage) Acetaminophen ----------------> N-acetyl-p-benzoquinone imine (NAPQI) (Glutathione) ---------------> NAPQI-glutathione conjugates ---------> Excretion 15. D 16. B 17. E 18. E
  • 32. 19. B 20. C – Botulinum causes paralysis of respiratory muscles. TOXICOLOGY MCQs SET 6
  • 33. MCQs Set 6 1 Cadmium is a highly toxic metal that causes__________. A Chemical pneumonitis B Damage to renal tubules C Cancer D GI tract irritation E All of the above 2 What is the main concern with the chlorinated hydrocarbon pesticide when compared with other classes of pesticide? A They are highly neurotoxic B They can cause death C They persist in the environment D They damage the crops E They can kill the cattle 3 Chlorine has toxic effects similar to _________. A Arsine B Phosgene C Mustard gas D Ammonia E Carbon monoxide 4 Which one of the following is not associated with sea food toxicity? A Ciguatoxin B Aflatoxin C Tetrodotoxin D Saxitoxin E Okadaic acid
  • 34. 5 What are the most likely toxic effects of Iodine? A Corrosive effects like that of chlorine B Oxidative damage to cellular molecules C Disturbed thyroid functions D Adrenal gland disturbances E Disturbed pancreatic gland functions 6 Ipecac syrup is derived from Cephaline ipecacuanha plant and used for ________. A Inducing emesis in case of oral poisoning B Supporting cardiac function in case of pesticide toxicity C To potentiate liver function D lung inflammatory diseases E GI tract cleaning 7 Which is the common target of toxicity of lithium, manganese and magnesium? A Cardiac muscles B Bones C Skeletal muscles D Central nervous system (CNS) E Immune system 8 Common toxicity target of elemental mercury and methyl mercury is ________. A Kidneys B Liver C Muscles D Skin E CNS 9 Common toxicity target of inorganic mercuric salts and organic mercury is ________. A Kidneys B Liver C Muscles D Skin E CNS 10 Common toxicity target of elemental mercury, inorganic mercuric salts and organic mercury is ________. A Kidneys B Liver C Muscles D Skin E CNS 11 Major rout of exposure to elemental mercury is _______. A Dermal B Inhalation C Ingestion D Intravenous E Intramuscular 12 ‘Metal fume fever’ is caused by inhalation of fumes of _______. A Zinc oxide B Elemental mercury C Chromium oxide D Ferric chloride E Any of the metal oxides
  • 35. 13 A 28 year old person accidentally consumed high doses of a nitrate. What sort of toxicity is expected in that person? A Reduced oxygen carrying capacity/hypoxia B Acute hypotension C Increased hypoxia and a little hypotension D Increased hypotension and a little hypoxia E Respiratory collapse 14 Toxicity targets of nonsteroidal anti-inflammatory drugs (NSAIDs) include_______. A CNS and muscles B Musculoskeletal system and liver C GI tract and kidneys D Cardiovascular system E Respiratory system 15 Morphine is obtained from the following plant A Nicotiana tabacum B Erythroxylum coca C Atropa belladonna D Papaver somniferum E Datura alba 16 Opioids like heroin and morphine can cause death by _________. A CNS depression B Cardiac failure C Respiratory depression and failure D Renal failure E Hemolytic anemia 17 Target organ of the herbicide paraquat is ________. A Liver B Kidney C Endocrine glands D Lung E Reproductive organs 18 Which of the following is not recommended in case of paraquat poisoning? A Charcoal administration B Oxygen administration C Washing of exposed skin D Maintenance of open airway E Fluid administration 19 What are the effects, other than the systemic effects of the smoke? A Thermal damage to the airways B Irritation C Asphyxia D A, B and C E B and C 20 Which of the following statement is associated with first order kinetic? A Excretion can be increased by increasing pH B Amount of excretion does not depend on the concentration of the toxicant C Amount of excretion can be increased by diuresis D Amount of excretion depends on the concentration of the toxicant E Excretion is not associated with the metabolism
  • 36. Answer keys MCQs Set 6: 1. E 2. C 3. D – Chlorine and ammonia, both have corrosive effects. 4. B – Aflatoxin is produced by a fugus , Aspergillus flavus, which usually contaminate peanuts. 5. C – Iodine is essential for the production of two thyroid hormones riiodothyronine (T3) and thyroxin (T4), which regulates the metabolism. Toxic doses of iodine disrupt this hormone system. 6. A 7. D 8. E 9. A 10. D 11. B 12. A 13. B – Nitrates cause vasodilation and reduces blood pressure. 14. C 15. D 16. C 17. D 18. B 19. D 20. D – In first order kinetics excretion is directly proportional to the concentration of the substance. TOXICOLOGY MCQs SET 7
  • 37. MCQs Set 7 1 Which one belongs to zero order kinetics? A Half-life remains constant even if the concentration of the compound is increased B Half-life is increased with the increasing concentration of the compound C Half-life is independent of the amount of compound D Half-life is indirectly proportional to the amount of excretion E Half-life is directly proportional to the amount of excretion 2 Wilson’s disease is associated with the accumulation of following metal- A Zinc B Calcium C Manganese D Iron E Copper 3 Which of the following favors the accumulation of the toxicants in biological system? A Molecular size and chemical properties B Hydrophilicity and protein binding C Resistance to metabolism and lipophilicity D Reuptake in the gut and enzyme inhibition E Entrance in the food chain and food web 4 Most of the copper in the body remains bound to __________ A Ceruloplasmin B Plasminogen C Albumin
  • 38. D Globulin E Collagen 5 Antidote/s administered in case of copper poisoning A Penicillamine B Disodium EDTA C Dimercaprol D A and B E A and C 6 Antidote for the arsenic poisoning A N-acetylcysteine B Sodium nitrate C Dimercaprol D EDTA E Fuller’s earth 7 Antidote for cyanide poisoning A N-acetylcysteine B Sodium nitrate C Dimercaprol D EDTA E Fuller’s earth 8 Antidote for paraquat A N-acetylcysteine B Sodium nitrate C Dimercaprol D EDTA E Fuller’s earth 9 Antidote for acetaminophen/paracetamol A N-acetylcysteine B Sodium nitrate C Dimercaprol D EDTA E Fuller’s earth 10 Therapeutic index is expressed as __________. A LD50/TD50 B LD50/ED50 C TD50/LD50 D TD50/ED99 E ED50/LD50 11 Margin of safety is expressed as __________. A LD1/ED99 B LD50/ED50 C TD50/LD50 D TD50/ED99 E ED50/LD50 12 What is the ‘First pass effect’? A When a larger part of the drug/toxicant bypasses the liver and reaches circulation. B When a larger part of the drug/toxicant first passes through the circulation and then reaches the liver for metabolism. C When a larger part of the drug/toxicant first gets converted into active metabolite in the circulation. D When a larger part of the drug/toxicant passes directly through urine without reaching to the liver. E When a larger part of the drug/toxicant is metabolized by GIT and/or liver before reaching to the circulation.
  • 39. 13 In the body most of the drugs remain bound to the plasma proteins. Which one of the following is the preferred bond? A Ionic bonds B Hydrogen bonds C Van der Waal’s forces D Hydrophobic interactions E All of the above 14 Xenobiotic metabolism will basically lead to _________. A Bioaccumulation of the xenobiotic B Altered chemical structure C Increased toxicity D Decreased toxicity E Increased renal excretion 15 Which is not a part of phase I biotransformation of toxicants? A Oxidation B Hydrolysis C Reduction D Glutathione conjugation E Epoxide hydrolase 16 Following are the reactions included in phase II biotransformation, except _________. A Glucuronidation B Quinone reduction C Sulfonation D Acetylation E Methylation 17 Which is the most important enzyme in xenobiotic biotransformation? A Epoxide hydrolase B Carboxylesterases C Flavin Monooxygenases D Cytochrome b5 E Cytochrome P450 18 Alcohol dehydrogenase, an important enzyme in biotransformation of alcohols, catalyzes the following type of reaction- A Reduction B Hydrolysis C Oxidation D Hydroxylation E Acetylation 19 Which one is the most common conjugation reaction in phase II biotransformation? A Glucuronidation B Sulfonation C Acetylation D Methylation E Glutathione conjugation 20 UDP–glucuronosyltransferases (UGTs), which catalyze the glucuronidation reactions in phase II biotransformation, are located in ______________ within the cell. A Cytoplasm B Endoplasmic reticulum C Mitochondria D Lysosomes E Peroxisomes
  • 40. Answer keys MCQs Set 7: 1. B – in zero order kinetics the rate of excretion remains constant, so increasing concentration means increase in half life. 2. E 3. C 4. A 5. E 6. C 7. B 8. E - Fuller’s earth works as an adsorbent and reduces the absorption of paraquat into the system. 9. A 10. B – lower value of therapeutic index (LD50/ED50) indicates higher risk of toxicity of therapeutic drugs. It is also expressed as TD50/ED50. (ED = Effective dose; LD = Lethal dose; TD = Toxic dose) 11. A 12. E 13. E 14. B - Xenobiotic metabolism basically alters the chemical structure in order to increase the exretion of the substance which ultimately may lead to increased or decreased toxicity. 15. D - Glutathione conjugation and other conjugation reactions are the part of Phase II biotransformation. 16. B 17. E 18. C 19. A 20. B TOXICOLOGY MCQs SET 8
  • 41. MCQs Set 8 1 Cytochrome P450 enzymes, which catalyze the largest portion of phase I biotransformation, are located in ______________ within the cell. A Cytoplasm B Endoplasmic reticulum C Mitochondria D Lysosomes E Peroxisomes 2 Bioactivation means _________________ A Activation of biological processes by means of drugs/toxicant exposure B Activation of a species after a long seasonal dormancy period C Conversion of a less- or non-toxic compound into a potentially toxic compound by metabolic process D Improvement of work efficiency using energetic nutrients E None of the above 3 If an enzyme is microsomal, it means it is __________ A Located in peroxisomes B Lysosomes C Endoplasmic reticulum D Golgi apparatus E Mitochondria
  • 42. 4 Sulfotransferases (SULT) catalyzes sulfonation reactions in Phase II. There are two major classes of these enzymes i) membrane bound and ii) soluble respectively located in _______ A Endoplasmic reticulum and cytoplasm B Mitochondria and extracellular matrix C Endoplasmic reticulum and extracellular matrix D Golgi apparatus and cytoplasm E Mitochondria and cytoplasm 5 Sulfonation involves the transfer of the following to the xenobiotic A SH‾ B SO4‾ C S2‾ D SO3‾ E SO42‾ 6 What is the cofactor in sulfonation reactions? A 3’-phosphoadenosine-5’-phosphosulfate (PAPS) B Sodium dodecyl sulfate C Adenosine triphosphate (ATP)-sulfurylase D Magnesium sulfate E Alkyl sulfonate 7 Which is the most common cofactor in one of the main phase II biotransformation reactions? A Uridine Diphosphate Glucose B Uridine Diphosphate Glucuronic Acid C Uridine Diphosphate Xylose D Uridine Diphosphate Galactose E All of the above 8 Which of the following is an example of bioactivation? A Acetaminophen → N-acetyl-p-benzoquinine imine B Malathion → Malaoxon C Aflatoxin → Aflatoxin-8,9 epoxide D Benzo(a)pyrene →benzo(a)pyrene 7,8-diol 9,10- epoxide E All of the above 9 Following are the examples of adverse effects of toxicants except _____. A Decreased lifespan B Increased incidences of limb injuries C Altered physiology and biochemistry D Behavioral instability E Decreased growth 10 ‘Ames test’ is used to detect _______. A Salmonella typhi infection. B Salmonella typhimurium infection. C Mutagenic potential of chemicals using in-vitro test with mutant strains of Salmonella typhimurium. D Carcinogenic potential of chemicals using in-vitro test with HeLa cancer cell lines. E Hepatotoxic potential of chemicals in cell cultures. 11 Anthracosis is a lung condition common in _______. A Gold miners B Diamond miners C People doing research on Bacillus anthracis D Coal miners E Wine distillers
  • 43. 12 Argyrosis is caused by _______ A mercury B cadmium C silver D lead E arsenic 13 Bagassosis disease is associated with the following organ. A Heart B Lung C Liver D Kidneys E Brain 14 The target organ of beryllium toxicity is _____. A Kidneys B Heart C Liver D Lung E Brain 15 Which of the following interface does not exist? A Blood-Brain Barrier B Blood-Testis Barrier C Blood-Follicle Barrier D Blood-Pancreas Barrier E Blood-Placenta Barrier 16 Workers involved in cotton processing may suffer with ______. A tuberculosis B silicosis C siderosis D stannosis E byssinosis 17 Ceiling Value (CV), a term used in occupational toxicology, denote_______ A Ceiling of a workplace should be well ventilated. B Concentration of an airborne toxicant in workplace. C Lethal concentration of a toxicant in workplace area. D level of heavy metal fumes reaching up to the ceiling. E none of the above. 18 Burning sensation after exposure to chilli pepper is an example of ________. A paresthesia B sensitization C receptor activation D contact dermatitis E chemesthesis 19 Chemosis is __________. A swelling of conjunctiva due to chemical exposure. B a lung condition after chemical inhalation. C occupational disease in chemical factory workers. D osmosis of chemicals. E inflammation of limbs after chemical exposure. 20 Ciguatera toxin is produced by _______. A Barracuda fish. B Pseudoalteromonas tetraodonis C Gambierdiscus toxicus D Fugu (Puffer fish) E Aspergillus flavus
  • 44. Answer keys MCQs Set 8: 1. B 2. C 3. C – In experimental set up endoplasmic reticulum are isolated using ultracentrifugation of the tissue homogenates. The endoplasmic reticulum cannot be collected in its natural form they are collected as small globular membrane bound forms known as microsomes, hence the enzymes found in endoplasmic reticulum are called microsomal enzymes. 4. D 5. D 6. A 7. B 8. E 9. B 10. C 11. D 12. C – greyish or dark grey pigmentation due to accumulation of silver in tissues. 13. B – Bagassosis is caused by inhalation of constituents of sugar cane. 14. D 15. D 16. E 17. B 18. E 19. A – It result in swelled surroundings of the eyes. 20. C – Barracuda fishes are found contaminated with ciguatera toxin. TOXICOLOGY MCQs SET 9
  • 45. MCQs Set 9 1 In ‘comet assay’ we measure the ________. A extent of hepatotoxicity. B extent of damage in spleen. C extent of damage in renal tubules. D extent of DNA damage in single cells. E pancreatic damage. 2 What is ‘bioavailability’? A Availability of biological entities for exposure to toxicants. B Ability of chemicals to be absorbed by organisms. C Chemicals in an approachable vicinity. D Availability of biologically produced agents. E None of the above. 3 Tooth enamel malformation is caused by _______ toxicity. A copper B phosphorus C fluoride D iron E lithium 4 Who discovered the carcinogenic effects of the chemicals? A Percivall Pott B Paracelsus C Orfilla D Bernardino Ramazzini E Socrates
  • 46. 5 Endotoxins are _________. A produced by yeasts B fungal toxin C bacterial toxins D produced by mycoplasmas E viral toxins 6 Endotoxins are released after death of some microorganisms. A True B False 7 ‘Environmental medicine’ encompasses the management of damages caused by toxicants to the environment. A True B False 8 In ‘first order kinetics’ reaction rate is indirectly proportional to the concentration of the substance. A True B False 9 ‘First pass effect’ is the biotransformation of chemicals in the intestine before reaching to the circulation. A True B False 10 Genotoxicity involves ________. A alteration in DNA sequence. B toxic damages to the DNA, such as DNA strand breaks. C Chromosomal abnormalities. D A & B E All of the above 11 Chemical carcinogenesis is not a toxic effect. A True B False 12 ‘Guinea-pig maximization test’ A used to asses skin carcinogens. B used to test maximum damage a toxicant can cause. C used to screen contact allergens. D used to assess penetration potential of cosmetics. E an alternative to LD50 experiment. 13 ‘Half-life’ and ‘Half time’ are different parameters. A True B False 14 ‘Hazard’ indicates inherent toxic properties of a substance, in other words a ‘source of danger’. A True B False 15 ‘Hormesis’ is a phenomenon where generally toxic/non-essential substances show beneficial effects at low doses. A True B False C maybe 16 An adverse condition is called ‘iatrogenic’ when it arises from lack of medical treatment. A True B False
  • 47. 17 When an individual show unusually high sensitivity towards a substance, the condition is known as _________. A Idiopathic B Icterus C Anaphylaxis D Idiosyncrasy E Hypersensitivity 18 ‘Micronucleus’ formation is an example of ________. A teratogenicity B carcinogenicity C genotoxicity D cytotoxicity E B & C 19 Apoptotic tissue injuries are accompanied by inflammation. A True B False 20 Substances in nanoforms (Nanosized Particles [NSPs], <100 nm) are less toxic than their crude/normal forms. A True B False Answer keys MCQs Set 9: 1. D 2. B 3. C 4. A 5. C – Endotoxins are integral part of cell walls of certain Gram-negative bacteria. They are also known as lipopolysaccharides (LPS). 6. A 7. B 8. B 9. B 10. E 11. B 12. C 13. B 14. A 15. A 16. B - An adverse condition is called ‘iatrogenic’ when it arises from medical treatment itself. 17. D 18. C 19. B – Necrotic injuries induce inflammation. 20. B
  • 49. MCQs Set 10 1 Oxidative stress is a condition where reactive oxygen species (ROSs) generation outweighs the body’s capacity to counter them. A True B False 2 Which of the following statements belongs to ‘toxicodynamics’? A Mercury vapors inhaled and accumulated in the CNS. B Binding of copper with ceruloplasmin. C Elimination of lead after chelation with dimercaprol. D Inhibition of ALAD enzyme in case of lead toxicity. E Urine levels of NAPQI-glutathione level. 3 A toxicant produced by an animal is called _________. A toxin B biotoxin C venom D poison E mycotoxin 4 An obese person survives a long period of exposure to a lipophilic toxicant T. After a certain period he pass through a period of starvation. What are the prospects in terms of toxicology? A He survived toxicant exposure, he will survive through starvation period as well. B He will be in good shape after the period is over. C Chances are there that he suffer severe toxicity by toxicant T which should be released from the fat tissue decomposing during starvation. D He has a lot of fat, so the starvation will not affect him. E None of the above. 5 A substance ‘S’ follows zero order kinetics. A person is exposed to a certain amount ‘A’ of ‘S’ on daily basis. The half-life of amount ‘A’ of ‘S’ is one day. The person was exposed to ‘S’ for three days. How much should he wait for complete elimination of the ‘S’? A 6 days B 5 days
  • 50. C 4 days D 3 days E 9 days 6 In Ames test a substance ‘M’ demonstrates significant mutagenic potential. What implications can be drawn from this information? A The substance ‘M’ is surly a carcinogen. B ‘M’ will induce cancer in animal models but not in humans. C ‘M’ may or may not have carcinogenic potential. D ‘M’ is an environmental contaminant. E Substance ‘M’ can be used as disinfectant in industrial and household set ups. 7 Gene mutations caused by chemicals initiates the process of carcinogenesis. Which kinds of genes are mutated in this process? A Oncogenes B Tumor suppressor genes C Protooncogenes D All of the above E B & C 8 Genotoxicity always leads to carcinogenesis/cancer. A True B False 9 DNA damage generally results in __________. A Cancer formation B Unregulated cell proliferation C Teratogenicity D Reproductive system damage E Cytotoxicity 10 Different agents of chemical, biological or physical origin have potential to alter DNA sequence and capable of causing cancer. What is your opinion regarding the above statement? A Only chemicals can cause cancer. B No physical agent is known to cause cancer. C Studies show that the agents of chemical, biological or physical origin can cause cancer. D The statement is not true. E Viruses do not belong to any category still they can cause certain types of cancer. 11 “All substances are poison….”. Under the aegis of this statement, when do you see the toxicity of table sugar? A In case of obesity and related heart problems. B Generation of extra heat and related discomfort during summers. C Dental caries. D Degeneration of different tissues/organs in case of diabetes mellitus. E Sugar is not a poison. 12 Use of performance enhancing drugs is illegal, but the drugs itself are not toxic to health at all. A True B False 13 Assume all the types of Ultra Violet (UV) rays (A, B and C) reaches down to the earth. Which of them all will be most damaging to the life? A UV C B UV A C UV B D UV A and C are equally damaging E UV A and B are equally damaging
  • 51. 14 Skin cancer is one of the major toxic effects of the UV exposure. What is/are the other major health effect/s? A Osteoporosis B Alopecia C Cataract D Hyperpigmentation E Contact dermatitis 15 Which one of the following is not an ionizing radiation? A X rays B infrared rays C Gamma rays D UV rays E Alpha rays 16 Chemical carcinogenesis has following stages (select all that applies). A Initiation B Immigration C Promotion D Metastasis E Progression 17 Which of the following is a non-solid tumor? A Mesothelioma B Renal cell carcinoma C leukemia D Basal cell carcinoma E Breast cancer 18 What is the common mechanism of action of most of the plant based products? A Hormesis B Antioxidant activity C Oxidative stress induction D Molecular mechanisms E B & D 19 Alternative medicine includes only herbal medicine. A True B False 20 Herbal formulations are always safe and do not need any toxicological evaluation and regulation. A False B True
  • 52. Answer keys MCQs Set 10: 1. A 2. D – Other statements are associated with ‘toxicokinetics’ where movement of toxicants is observed in order to study what body does to the toxicants. 3. C 4. C 5. D 6. C 7. E 8. B 9. E 10. C 11. D 12. B 13. A – UV C has shortest wave length and highest energy level in UV group and, UV B comes second and the UV A is third in the order. However, in natural conditions only UV A reaches to the ground in significant amount, which is damaging to the life. Ozone layer filters out UV C completely and UV B partially. 14. C 15. B 16. A,C,D,E 17. C 18. E 19. B 20. A Bibliography
  • 53. Bibliography • A small Dose of Toxicology (Second Edition). Steven G. Gilbert. Healthy World Press, US, 2012. • A text book of modern toxicology (Third Edition). Ernest Hodgson (Ed) John Wiley & Sons, Inc., Hoboken, New Jersey, 2004. • Casarett and Doull’s Toxicology: The Basic Science of Poisons (Seventh Edition). Curtis D. Klaassen (Ed). McGraw-Hill, New York, 2008. • Lu’s Basic Toxicology: Fundamentals, Target Organs, and Risk Assessment (Fifth Edition). Lu FC, and Kacew S. Informa Healthcare USA, Inc., New York, 2009. • Principles Of Toxicology: Environmental and Industrial Applications (Second Edition). Williams PL, James RC, Roberts SM (Eds). John Wiley & Sons, New York, 2000. • Glossary Of Terms Used In Toxicology, 2nd Edition (IUPAC Recommendations 2007), International Union Of Pure And Applied Chemistry, Chemistry And Human Health Division. Duffus HS, Nordberg M and Templeton DM. Pure and Applied Chemistry, Vol.79, No.7, pp.1153–1344, 2007. View publication stats View publication stats